Blackboard EMQs Endo Flashcards

1
Q

An 19 year old girl wearing loose clothes comes to see you in your surgery. She is worried because all her friends have started their periods but she hasn’t. On direct questioning she refuses to comment about her eating habits & says that she has put on weight. Her current BMI is 14.

Match the options with the appropriate diagnosis.
A.	Bullimia nervosa
B.	Pregnancy
C.	Obesity
D.	Grave's disease 
E.	Colonic malignancy
F.	Cushing's syndrome
G.	Anorexia nervosa
H.	Toxic solitary adenoma
I.	Hypothyroidism
J.	PCOS
K.	Bronchial carcinoma
A

G. Anorexia nervosa

Anorexia nervosa often occurs in women which is a key risk factor. Puberty/adolescence and obsessive personality traits are additional risks. There is a higher incidence reported in western cultures (presumably skinny models on TV, adverts etc…) and studies on immigrants moving to a western culture exhibited a higher incidence. There is also postulated to be a genetic contribution from twin studies.Those who in higher socioeconomic classes are more affected. BMI is usually <17.5 and the patient often has a fear of weight gain and will refuse intervention to gain weight. The patient often fails to acknowledge how thin they are and can indicate how some parts of her body are ‘big’. Amenorrhoea is also a common complaint. Often anorexics eat vegetarian or vegan diets and you should inquire about the patient’s eating habits.There are two subtypes of AN. Restrictive where the patient will diet and exercise or the bingeing/purging types where there is also calorie restriction but ocassional binges and purging afterwards. There is also a tendency to laxative, diuretic and enema abuse.

On examination, the patient may be wearing baggy clothing to conceal their weight. Low fat may lead to protruding bony structures like the scapula and there may be lanugo hair (fine body hair). Signs of vomiting include erosion of enamel and swollen salivary glands, and occassionally scars on the back of the hand (Russell’s sign). The mainstay of treatment is the creation of a structured eating plan with the aim of gaining weight gradually with some form of psychotherapy.

Bulimia nervosa (again, more prevalent in women) is characterised on the other hand by recurrent episodes of binge eating followed by compensatory behaviour (for example, self-induced vomiting, exercise and laxatives) to compensate for the binge. There is a sense of a lack of control during the binge. Patients are usually physically normal. The best treatment for bulimia is considered to be CBT where available.

How well did you know this?
1
Not at all
2
3
4
5
Perfectly
2
Q

A 78 year old gentleman with a 40 pack year history of smoking cigarettes presents to your surgery saying that his cough has changed in the last 3 weeks. He’s now coughing all through the day. On direct questioning he admits to having to buy trousers 3 inches smaller than he usually does.

Match the options with the appropriate diagnosis.
A.	Bullimia nervosa
B.	Pregnancy
C.	Obesity
D.	Grave's disease 
E.	Colonic malignancy
F.	Cushing's syndrome
G.	Anorexia nervosa
H.	Toxic solitary adenoma
I.	Hypothyroidism
J.	PCOS
K.	Bronchial carcinoma
A

K. Bronchial carcinoma

The 40 pack year history of smoking and weight loss point to a bronchial carcinoma. Initial investigation is with a CXR. Diagnosis relies on pathological confirmation from a tissue sample, often obtained from bronchoscopy. First line treatment aims at surgical resection if possible. Small cell lung cancer is treated with chemotherapy and is associated with SIADH and ectopic ACTH. Non-small cell lung cancer is more often associated with clubbing. Squamous cell carcinoma is associated with PTHrp release and is treated with radiotherapy. Adenocarcinomas are usually located peripherally in the lung and are more common in non-smokers although most cases are still associated with smoking. The paraneoplastic syndromes may include Lambert-Eaton myasthenic syndrome.

How well did you know this?
1
Not at all
2
3
4
5
Perfectly
3
Q

A 38 year old female complains of feeling tired all the time. She says that because of this she has put on weight & has been feeling depressed. On examination you note that she is bradycardic, has slow-relaxing reflexes & takes a little while longer to respond to commands.

Match the options with the appropriate diagnosis.
A.	Bullimia nervosa
B.	Pregnancy
C.	Obesity
D.	Grave's disease 
E.	Colonic malignancy
F.	Cushing's syndrome
G.	Anorexia nervosa
H.	Toxic solitary adenoma
I.	Hypothyroidism
J.	PCOS
K.	Bronchial carcinoma
A

I. Hypothyroidism

This patient has hypothyroidism. Worldwide, the most common cause is iodine deficiency. Other causes include Hashimoto’s or secondary and tertiary hypothyroidism. It can also result from viral de Quervain’s thyroiditis or postpartum thyroiditis. Symptoms include those mentioned (depression, fatigue, weight gain, bradycardia and sluggish reflexes) as well as others such as constipation, cold intolerance, menstrual problems in females, dry skin and muscle cramps. Diagnosis is based on measurement of TSH and thyroid hormones. Treatment is by replacement of T4 with or without T3 in combination. If the patient has normal T3 and T4 but mildly elevated TSH, this is described as subclinical hypothyroidism.

How well did you know this?
1
Not at all
2
3
4
5
Perfectly
4
Q

A 34 year old lady with amenorrhoea comes to see you because she has put on weight in the last 4 months. She says that it is not a lot, but she has had strange desires to eat plain starchy foods & that strong smelling foods make her nauseous.

Match the options with the appropriate diagnosis.
A.	Bullimia nervosa
B.	Pregnancy
C.	Obesity
D.	Grave's disease 
E.	Colonic malignancy
F.	Cushing's syndrome
G.	Anorexia nervosa
H.	Toxic solitary adenoma
I.	Hypothyroidism
J.	PCOS
K.	Bronchial carcinoma
A

B. Pregnancy

This patient is pregnant. Always consider this in a presentation of amenorrhoea. In terms of the terminology you may come across in EMQs: a woman who has never been pregnant is a nulligravida, a woman who is pregnant for the first time is a primigravida and a woman in subsequent pregnancies is multiparous. Most women experience symptoms such as cravings for certain foods, nocturia, N&V and tiredness. The weight gain in this question is also a give away. There are also numerous signs associated with pregnancy – you can look these up if you are interested. Home pregnancy tests looks at urinary beta hCG. Some women may not realise they are pregnany for quite a long time and it may be difficult to tell in those who are obese.

How well did you know this?
1
Not at all
2
3
4
5
Perfectly
5
Q

A slim 29 year old lady wearing a T-shirt & skirt comes to see you in January. She was wondering what the lump was on one side of her neck. On direct questioning she says her periods have been heavy & irregular. On examination you note that she is tachycardic & has a tremor, but has no abnormalities of her eyes.

Match the options with the appropriate diagnosis.
A.	Bullimia nervosa
B.	Pregnancy
C.	Obesity
D.	Grave's disease 
E.	Colonic malignancy
F.	Cushing's syndrome
G.	Anorexia nervosa
H.	Toxic solitary adenoma
I.	Hypothyroidism
J.	PCOS
K.	Bronchial carcinoma
A

H. Toxic solitary adenoma

This is a case of toxic solitary adenoma (a ‘hot’ adenoma). It typically presents with a large nodule in the thyroid gland and hyperthyroidism (in which case the nodule is typically >3cm). TSH is an effective screening test, which will be suppressed in a toxic adenoma (think about the thyroid axis). A thyroid scan shows a hot area and suppression of extranodular thyroid tissue. The hyperthyroidism generally does not remit and definitive treatment is most often with radioactive iodine (I-131). If the patient is pregnant, antithyroid drugs are first line and management should incorporate an MDT ethos. A key risk factor is iodine deficiency. Toxic adenomas are most common in ages 20-40 and radiation and female gender may be additional risk factors. There are no stigmata of Graves’ disease present such as eye abnormalities.

How well did you know this?
1
Not at all
2
3
4
5
Perfectly
6
Q

A 62 year old male heavy, long term smoker presenting with infection of a wound which refuses to heal.

Match the investigation results with the patient.
A.	Low ACTH, normal cortisol
B.	High ACTH, high cortisol
C.	High ACTH, low cortisol
D.	Low ACTH, high cortisol
E.	Low ACTH, low cortisol
F.	High ACTH, normal cortisol
G.	Normal ACTH, high cortisol
A

B. High ACTH, high cortisol

This wound is refusing to heal because of the high cortisol level, which is as a result of elevated ACTH due to a small cell lung cancer producing it ectopically as part of a paraneoplastic syndrome. Hence, ACTH and cortisol are both high here.

How well did you know this?
1
Not at all
2
3
4
5
Perfectly
7
Q

A 72 year old lady under long term therapy with biannual bone density assessment for rheumatoid arthritis.

Match the investigation results with the patient.
A.	Low ACTH, normal cortisol
B.	High ACTH, high cortisol
C.	High ACTH, low cortisol
D.	Low ACTH, high cortisol
E.	Low ACTH, low cortisol
F.	High ACTH, normal cortisol
G.	Normal ACTH, high cortisol
A

E. Low ACTH, low cortisol

This patient is on steroids for RA (hence the biannual bone density assessment). The anchor drug for RA is methotrexate which is the most commonly used DMARD. Other DMARDs include leflunomide, sulphasalazine and hydroxychloroquine. Corticosteroids given for RA will suppress the secretion of ACTH by the pituitary gland and as a result cause cortisol to become low.

How well did you know this?
1
Not at all
2
3
4
5
Perfectly
8
Q

An 18 year old woman with a bitemporal hemianopia, tiredness, hypoglycaemia and low BP and postural hypotension.

Match the investigation results with the patient.
A.	Low ACTH, normal cortisol
B.	High ACTH, high cortisol
C.	High ACTH, low cortisol
D.	Low ACTH, high cortisol
E.	Low ACTH, low cortisol
F.	High ACTH, normal cortisol
G.	Normal ACTH, high cortisol
A

E. Low ACTH, low cortisol

The bitemporal hemianopia suggests a pituitary adenoma compressing the optic chiasm. The patient presents with signs and symptoms of adrenal insufficiency indicating low cortisol. The cause is the pituitary adenoma (this is secondary adrenal insufficiency) leading to low ACTH.

How well did you know this?
1
Not at all
2
3
4
5
Perfectly
9
Q

A 43 year old woman who initially presented with a bilateral hemianopia & has gone on to develop Cushingoid signs.

Match the investigation results with the patient.
A.	Low ACTH, normal cortisol
B.	High ACTH, high cortisol
C.	High ACTH, low cortisol
D.	Low ACTH, high cortisol
E.	Low ACTH, low cortisol
F.	High ACTH, normal cortisol
G.	Normal ACTH, high cortisol
A

B. High ACTH, high cortisol

A pituitary adenoma here is secreting ACTH. This is Cushing’s disease responsible for most cases of Cushing’s syndrome. Hence, ACTH is high and so is cortisol.

How well did you know this?
1
Not at all
2
3
4
5
Perfectly
10
Q

A 55 year old man, whose abdominal CT shows an adrenal adenoma.

Match the investigation results with the patient.
A.	Low ACTH, normal cortisol
B.	High ACTH, high cortisol
C.	High ACTH, low cortisol
D.	Low ACTH, high cortisol
E.	Low ACTH, low cortisol
F.	High ACTH, normal cortisol
G.	Normal ACTH, high cortisol
A

D. Low ACTH, high cortisol

This is adrenal Cushing’s where an excess of cortisol is being produced by the adrenal gland adenoma. This excess cortisol suppresses ACTH production.

How well did you know this?
1
Not at all
2
3
4
5
Perfectly
11
Q

A 55 year old Asian man complaining of nocturia. Random blood glucose 10.2mmol/l. He is overweight.

Select the most appropriate treatment from the list of options.
A.	Statin
B.	Intravenous dextrose
C.	Laser treatment
D.	Insulin
E.	Oral glucose or sugar
F.	Diet alone
G.	Blood pressure control
H.	Oral hypoglycaemic drug
I.	Bed rest
A

F. Diet alone

This patient is symptomatic and has a random blood glucose of 10.2. This patient does not quite meet the diagnostic criteria for DM. Symptomatic patients need a single random blood glucose of >11.1 or single fasting glucose of >7. Asymptomatic patients need two separate elevated readings for a diagnosis. Alternatively if there are borderline results, an OGTT can be conducted to see if plasma glucose is raised >11.1 two hours after an oral glucose load of 75g. A patient is said to have impaired fasting glucose if fasting glucose falls between 6.1-6.9. Impaired glucose tolerance is present if plasma glucose 2 hours after oral glucose load in OGTT falls between 7.8-11.0. First line intervention in this situation, and in newly diagnosed DM is diet and lifestyle advice and changes.

How well did you know this?
1
Not at all
2
3
4
5
Perfectly
12
Q

A 47 year old man with diabetes for 10 years. At review BP 130/80, glycosylated haemoglobin 8.2% (normal <6.5%), plasma cholesterol 5.7mmol/l (normal 5.2-6.5mmol/l), despite careful dietary intervention

Select the most appropriate treatment from the list of options.
A.	Statin
B.	Intravenous dextrose
C.	Laser treatment
D.	Insulin
E.	Oral glucose or sugar
F.	Diet alone
G.	Blood pressure control
H.	Oral hypoglycaemic drug
I.	Bed rest
A

H. Oral hypoglycaemic drug

Careful dietary intervention has failed to keep HbA1c in check so the patient will need to be started on an oral hypoglycaemic drug. First line is metformin. It is worth noting that HbA1c values, since June 2011, are no longer expressed as a percentage and are now given in mmol/mol. HbA1c is glycated Hb and provides an estimation of glycaemic control over the life span of red blood cells (around 60 days). Fructosamine is measured instead if there is a Hb disorder or RBC life span is decreased as HbA1c is only reliable if normal Hb is present with normal RBC life spans. Fructosamine is a glycated plasma protein which provides information on glucose levels over the previous 1-3 weeks.

How well did you know this?
1
Not at all
2
3
4
5
Perfectly
13
Q

A 52 year old man with diabetes for 15 years. Recently found to have microalbuminuria. Glycosylated haemoglobin 7.2%. BP 150/85. Cholesterol 5.2mmol/l.

Select the most appropriate treatment from the list of options.
A.	Statin
B.	Intravenous dextrose
C.	Laser treatment
D.	Insulin
E.	Oral glucose or sugar
F.	Diet alone
G.	Blood pressure control
H.	Oral hypoglycaemic drug
I.	Bed rest
A

G. Blood pressure control

BP control with an ACE inhibitor is necessary to reduce progression of diabetic nephropathy. Microalbuminuria is the earliest detectable sign and indicates trace amounts of albumin not detectable with standard urinalysis. If untreated, microalbuminuria will progress to intermittent albuminuria and then to persistent albuminuria which is 5-10 years away from ESRF. The main contributor to the development of diabetic nephropathy is hypertension. Other contributors include poor glycaemic control and renovascular disease such as renal artery stenosis. It is important to test kidney function and urine regularly

How well did you know this?
1
Not at all
2
3
4
5
Perfectly
14
Q

A 55 year old woman in diabetic clinic. She has been waiting for 2 hours & starts feeling sweaty & weak

Select the most appropriate treatment from the list of options.
A.	Statin
B.	Intravenous dextrose
C.	Laser treatment
D.	Insulin
E.	Oral glucose or sugar
F.	Diet alone
G.	Blood pressure control
H.	Oral hypoglycaemic drug
I.	Bed rest
A

E. Oral glucose or sugar

This patient has symptoms of hypoglycaemia, present when glucose drops <3mmol/L. Symptoms include sweating, weakness, drowsiness, palpitations and anxiety. This is a mild case and can be managed with oral glucose or sugar. Patients with either long standing DM or on beta blockers may become unaware of hypoglycaemia and become profoundly hypoglycaemic before symptoms develop. In DM, hypoglycaemia is usually secondary to insulin or oral hypoglycaemics. Non DM causes of hypoglycaemia include insulinomas, alcohol, liver failure and Addison’s disease.

How well did you know this?
1
Not at all
2
3
4
5
Perfectly
15
Q

A 22 year old medical student notices that he is thirsty & drinking 3-4 litres of fluid a day while studying for finals. He tests his urine & finds glucose 3+ & ketones 2+.

Select the most appropriate treatment from the list of options.
A.	Statin
B.	Intravenous dextrose
C.	Laser treatment
D.	Insulin
E.	Oral glucose or sugar
F.	Diet alone
G.	Blood pressure control
H.	Oral hypoglycaemic drug
I.	Bed rest
A

D. Insulin

This young person has presented with T1DM, a condition characterised by absolute insulin deficiency. As a result, insulin treatment is required to prevent DKA and chronic complications of hyperglycaemia. The presentation is most often with polyuria, polydipsia, weight loss and weakness. Ketonuria in the presence of hyperglycaemia suggests T1DM. Ketonuria occurs due to the fact the body has no insulin to drive glucose into the cells and hence ketone bodies are utilised by the brain (fatty acids cannot cross the BBB). Diet, exercise and insulin therapy is crucial in T1DM.

How well did you know this?
1
Not at all
2
3
4
5
Perfectly
16
Q

45 year old man with 20 year history of diabetes. For the last few months he has had a burning sensation in his feet.

Select the most likely situation that is described by the following case scenarios.
A.	 Autonomic neuropathy
B.	Retinopathy
C.	Peripheral neuropathy
D.	Hyperosmolar hyperglycaemic state
E.	Ischaemic heart disease
F.	Cataract
G.	Peripheral arterial disease
H.	Onset type 2 diabetes
I.	Necrobiosis lipoidica
J.	Gastroparesis
K.	Ketoacidosis
L.	Onset type 1 diabetes
M.	Nephropathy
N.	Hypoglycaemia
A

C. Peripheral neuropathy

This is a case of diabetic peripheral neuropathy. This is a microvascular complication of DM and is characterised by peripheral nerve dysfunction. Pain is a common complaint such as the burning sensation this patient describes. Patient’s may also describe the pain as prickling or sticking. Complications range from the painless neuropathic ulcer at areas of the foot where there is weight loading (particularly the metatarsal heads), to the Charcot foot with severe architectural destruction of the foot. Foot ulceration is a common precusor to amputation. Foot care is crucial in DM. Examination should include peripheral pulses, reflexes and sensation to light touch with a 10g monofilament, vibration (128Hz tuning fork), pinprick and proprioception. The pain may be treated with medications like pregabalin and gabapentin.

How well did you know this?
1
Not at all
2
3
4
5
Perfectly
17
Q

72 year old man with diabetes. He has been on insulin for 6 years. His daughter had noticed that he was increasingly vague. He was found unconscious. BP 160/90. Plasma urea, creatinine & electrolytes were normal.

Select the most likely situation that is described by the following case scenarios.
A.	 Autonomic neuropathy
B.	Retinopathy
C.	Peripheral neuropathy
D.	Hyperosmolar hyperglycaemic state
E.	Ischaemic heart disease
F.	Cataract
G.	Peripheral arterial disease
H.	Onset type 2 diabetes
I.	Necrobiosis lipoidica
J.	Gastroparesis
K.	Ketoacidosis
L.	Onset type 1 diabetes
M.	Nephropathy
N.	Hypoglycaemia
A

N. Hypoglycaemia

This patient has symptoms of hypoglycaemia, present when glucose drops <3mmol/L. Symptoms include sweating, weakness, drowsiness, palpitations and anxiety. LOC can occur. Patients with either long standing DM or on beta blockers may become unaware of hypoglycaemia and become profoundly hypoglycaemic before symptoms develop. In DM, hypoglycaemia is usually secondary to insulin or oral hypoglycaemics. Non DM causes of hypoglycaemia include insulinomas, alcohol, liver failure and Addison’s disease. How would you manage this patient?

How well did you know this?
1
Not at all
2
3
4
5
Perfectly
18
Q

26 year old Asian man is found to have glycosuria. He has been waking up at night to pass urine for 2 months. His father & uncle are both diabetic.

Select the most likely situation that is described by the following case scenarios.
A.	 Autonomic neuropathy
B.	Retinopathy
C.	Peripheral neuropathy
D.	Hyperosmolar hyperglycaemic state
E.	Ischaemic heart disease
F.	Cataract
G.	Peripheral arterial disease
H.	Onset type 2 diabetes
I.	Necrobiosis lipoidica
J.	Gastroparesis
K.	Ketoacidosis
L.	Onset type 1 diabetes
M.	Nephropathy
N.	Hypoglycaemia
A

H. Onset type 2 diabetes

The patient’s symptoms and FH argue for type 2 diabetes. There is a significant genetic component in T2DM. Onset is generally preceded by a period of insulin resistance and hyperinsulinaemia before high glucose becomes detectable. There is a high concurrent risk of obesity, dyslipidaemia and hypertension. Lifestyle changes and metformin are indicated in most patients. BP and lipid management are also needed to reduce macrovascular risk and improve prognosis. BP and blood sugars should be managed to reduce the risk of microvascular complications such as retinopathy and nephropathy.

How well did you know this?
1
Not at all
2
3
4
5
Perfectly
19
Q

32 year old woman who has been diabetic for 25 years. She can no longer cross a road on her own as she is worried that she may collapse.

Select the most likely situation that is described by the following case scenarios.
A.	 Autonomic neuropathy
B.	Retinopathy
C.	Peripheral neuropathy
D.	Hyperosmolar hyperglycaemic state
E.	Ischaemic heart disease
F.	Cataract
G.	Peripheral arterial disease
H.	Onset type 2 diabetes
I.	Necrobiosis lipoidica
J.	Gastroparesis
K.	Ketoacidosis
L.	Onset type 1 diabetes
M.	Nephropathy
N.	Hypoglycaemia
A

A. Autonomic neuropathy

This is diabetic autonomic neuropathy which typically occurs later in the disease course. Patients have a reduced response in HR and BP and postural hypotension occurs due to efferent sympathetic denervation (reduced vasoconstriction of peripheral vascular beds). Patients can feel week, faint, dizzy and light headed and may faint on standing. There may also be GI symptoms from vagal dysfunction. Bladder dysfunction and erectile dysfunction can also occur. Women can complain of a reduced libido and increased pain during sex.

How well did you know this?
1
Not at all
2
3
4
5
Perfectly
20
Q

56 year old man with newly diagnosed diabetes is given laser treatment.

Select the most likely situation that is described by the following case scenarios.
A.	 Autonomic neuropathy
B.	Retinopathy
C.	Peripheral neuropathy
D.	Hyperosmolar hyperglycaemic state
E.	Ischaemic heart disease
F.	Cataract
G.	Peripheral arterial disease
H.	Onset type 2 diabetes
I.	Necrobiosis lipoidica
J.	Gastroparesis
K.	Ketoacidosis
L.	Onset type 1 diabetes
M.	Nephropathy
N.	Hypoglycaemia
A

B. Retinopathy

This patient has a microvascular complication of diabetes – diabetic retinopathy, for which he has had retinal photocoagulation with a laser. This suggests either macular involvement or proliferative diabetic retinopathy which has necessitated either panretinal laser therapy or macular laser therapy. Diabetic retinopathy, if suspected, is diagnosed on photographing the fundus with digital methods (preferred over a fundoscope). Microaneurysms, cotton wool spots, haemorrhages, hard/lipid exudates and neovascularisation may all be seen depending on the stage. Young onset T1DM is more associated with retinopathy, and additional risks include a longer duration of DM and poor glycaemic control compounded by renal disease and hypertension. There are also ethnic variations in risk.

How well did you know this?
1
Not at all
2
3
4
5
Perfectly
21
Q

A 60 year old diabetic lady visits her GP complaining of tingling in her hands. This is worse at night. She describes numbness, and if there is feeling she describes tingling. This is in both hands only; not in her arms.

Match the description to the relevant diagnosis.
A.	Abducens nerve palsy
B.	Occulomotor nerve palsy
C.	Commmon peroneal nerve palsy 
D.	Symmetrical sensory polyneuropathy
E.	Sciatica
F.	Diabetic amyotrophy
G.	Amaurosis fugax
H.	Charcot joint
I.	Carpal tunnel syndrome
J.	Ulnar nerve palsy
K.	Retinopathy
A

D. Symmetrical sensory polyneuropathy

Diabetic neuropathy can be autonomic or peripheral. This is a case of diabetic peripheral sensory neuropathy. This is a microvascular complication of DM and is characterised by peripheral nerve dysfunction. The tingling this patient describes is a common complaint and loss of sensation typically occurs in a symmetrical ‘glove and stocking’ distribution. Patient’s may also describe a pain or unpleasant sensation which is prickling, burning or sticking. Examination should include peripheral pulses, reflexes and sensation to light touch, vibration (128Hz tuning fork), pinprick and proprioception. Any pain can be treated with medications like gabapentin.

How well did you know this?
1
Not at all
2
3
4
5
Perfectly
22
Q

A 73 year old, previously fit male presents with difficulty ascending stairs. Abnormalities noted on examination are weakness of knee flexion, which is more pronounced on the left with some wasting of the quadriceps & diminished knee reflexes. He is noted to have glycosuria.

Match the description to the relevant diagnosis.
A.	Abducens nerve palsy
B.	Occulomotor nerve palsy
C.	Commmon peroneal nerve palsy 
D.	Symmetrical sensory polyneuropathy
E.	Sciatica
F.	Diabetic amyotrophy
G.	Amaurosis fugax
H.	Charcot joint
I.	Carpal tunnel syndrome
J.	Ulnar nerve palsy
K.	Retinopathy
A

F. Diabetic amyotrophy

Diabetic amyotrophy, more common in T2DM, is an uncommon peripheral diabetic neuropathic complaint. It presents with severe muscle weakness and pain with proximal thigh muscle atrophy. This patient’s glycosuria suggests undiagnosed T2DM, which in any case, you can guess he has given the question stem is called ‘Diabetic Complications’. The weak knee flexion and quadriceps wasting is typical of diabetic amyotrophy. The reduced reflexes are another sign of peripheral neuropathy.

How well did you know this?
1
Not at all
2
3
4
5
Perfectly
23
Q

A 62 year old male diabetic presents with a bad limp. His wife has forced him to come as he can no longer manage well upstairs. He denies any pain in any joints or the legs and blames old age. You notice that his shoes do not seem to fit well. OE you find that his shoes do not seem to fit well. OE you find that the joint is red, swollen and looks misaligned.

Match the description to the relevant diagnosis.
A.	Abducens nerve palsy
B.	Occulomotor nerve palsy
C.	Commmon peroneal nerve palsy 
D.	Symmetrical sensory polyneuropathy
E.	Sciatica
F.	Diabetic amyotrophy
G.	Amaurosis fugax
H.	Charcot joint
I.	Carpal tunnel syndrome
J.	Ulnar nerve palsy
K.	Retinopathy
A

H. Charcot foot

The Charcot foot, not to be confused with Charcot’s disease, occurs in about 1% of diabetics and is a progressive condition marked by severe architectural destruction of the foot. Go have a look at some photos. There are thought to be multiple aetiological factors from the loss of sensation peripherally to increased blood flow to the foot as a result of autonomic dysfunction.

How well did you know this?
1
Not at all
2
3
4
5
Perfectly
24
Q

A 66 year old male with type 2 diabetes complains of episodes of loss of vision in the right eye that may last up to 2 hours. On examination he is noted to have an irregularly irregular pulse of 70 beats per minute & a blood pressure of 155/95. Fundoscopy is normal

Match the description to the relevant diagnosis.
A.	Abducens nerve palsy
B.	Occulomotor nerve palsy
C.	Commmon peroneal nerve palsy 
D.	Symmetrical sensory polyneuropathy
E.	Sciatica
F.	Diabetic amyotrophy
G.	Amaurosis fugax
H.	Charcot joint
I.	Carpal tunnel syndrome
J.	Ulnar nerve palsy
K.	Retinopathy
A

G. Amaurosis fugax

Amaurosis fugax is a transient and painless loss of vision in one eye due to the passage of an embolus into the central retinal artery. The AF has thrown off a cardiac embolus which has passed into the central retinal artery (resulting in a temporary loss of blood flow to the retina and hence loss of vision). This patient needs to be started on aspirin at once while a definitive treatment of the underlying aetiology is sought (treatment of AF).

How well did you know this?
1
Not at all
2
3
4
5
Perfectly
25
Q

Mrs Y describes numbness and paresthesia in her right hand. She describes this as “coming on gradually over months”. The tingling wakes her at night and she has tried changing sleeping positions to no avail. She has come to the doctor now as her hand is becoming weak and she has dropped mugs several times.

Match the description to the relevant diagnosis.
A.	Abducens nerve palsy
B.	Occulomotor nerve palsy
C.	Commmon peroneal nerve palsy 
D.	Symmetrical sensory polyneuropathy
E.	Sciatica
F.	Diabetic amyotrophy
G.	Amaurosis fugax
H.	Charcot joint
I.	Carpal tunnel syndrome
J.	Ulnar nerve palsy
K.	Retinopathy
A

I. Carpal tunnel syndrome

Carpal tunnel syndrome is the most common nerve entrapment and women just past middle age are at the highest risk. Symptoms include numbness/tingling of the thumb and radial fingers, an aching wrist and clumsiness (especially with fine motor tasks). The symptoms are of gradual onset and often wake the patient up at night, and is relieved by shaking the wrist. Numbness is normally on the palmar aspect of the thumb, index and middle fingers (but not the little finger). When the patient wakes up, there may be difficulty flexing or extending fingers. Symptoms in the day tend to be associated with activity. The most sensitive and specific test for diagnosis is EMG and can confirm damage to the median nerve in the carpal tunnel and categorise the severity of the damage. There are specific tests for CTS such as Tinel’s test and Phalen’s test, though clinically these are not particularly useful due to sensitivity and specificity.

CTS is caused by anything that causes a reduction in the size of the carpal tunnel – from inflammation, arthritis and tenosynovitis to old fractures. In CTS there is preserved sensation of the palm as the palmar cutaneous branch comes off a few cm above the carpal tunnel.

Remember your anatomy in 2nd year? Probably not… but the median nerve lies in the carpal tunnel, deep to the flexor retinaculum and in the concavity bounded by the carpal bones. Other structures lying in the carpal tunnel include FPL, FDS and FDP.

How well did you know this?
1
Not at all
2
3
4
5
Perfectly
26
Q

80 year old presents with symptoms of dysphagia. He has been a lifelong smoker. On examination there is a reducible mass over lateral aspect of the neck.

Choose the most likely diagnosis from the list of options.
A.	Thyroid goitre
B.	Thyroiditis
C.	TB lymphadenitis
D.	Thyroglossal cyst
E.	Pharyngeal pouch
F.	Salivary gland tumour
G.	Lymphadenopathy secondary to infection
H.	Glandular fever
I.	Carotid body tumour
J.	Parotitis
K.	Mumps
L.	Lymphadenopathy secondary to malignancy
M.	Hodgkin's lymphoma
N.	Thyroid cancer
A

E. Pharyngeal pouch

A pharyngeal pouch is also called a Zenker’s diverticulum (it is a false diverticulum – i.e. it does not involve all layers of the oesophageal wall). The dehiscence of Killian lies in the posterior laryngopharynx which is where the wall herniates through giving an outpouching just above the cricopharyngeal muscle. It may help to gave a look at a picture in an anatomy textbook. A pharyngeal pouch can cause symptoms of dysphagia and the sensation of a lump in the neck. There may also be regurgitation of food, cough, halitosis and gurgling noises and the condition is associated with webs. Additionally, it may be asymptomatic. A barium swallow will confirm the dagnosis. Surgical intervetion may be necessary.

How well did you know this?
1
Not at all
2
3
4
5
Perfectly
27
Q

An anxious 19 year old female presents with a lump in the neck. She has lost 3kg in 3 months. On examination there is lymphadenopathy on both sides of the neck & larger nodes on the right. Her pulse is 96 regular; thyroid function tests are normal.

Choose the most likely diagnosis from the list of options.
A.	Thyroid goitre
B.	Thyroiditis
C.	TB lymphadenitis
D.	Thyroglossal cyst
E.	Pharyngeal pouch
F.	Salivary gland tumour
G.	Lymphadenopathy secondary to infection
H.	Glandular fever
I.	Carotid body tumour
J.	Parotitis
K.	Mumps
L.	Lymphadenopathy secondary to malignancy
M.	Hodgkin's lymphoma
N.	Thyroid cancer
A

M. Hodgkin’s lymphoma

The history of weight loss here along with cervical lymphadenopathy point to lymphoma. Hodgkin’s is localised to a single group of nodes (normally the cervical and/or supraclavicular) and extranodal involvement is rare. Mediastinal involvement is common. Spread is contiguous and B symptoms may be present such as a low grade fever, weight loss and night sweats. Pruritis may be found in approximately 10% of cases but has no prognostic significance. 50% of cases is associated with EBV infection and distribution is bimodal with peaks in young and old. There is classically pain in lymph nodes on alcohol consumption. While this question does not specifically scream out Hodgkin’s (though the pattern of involvement makes it more likely), the question is not complicated by the option of NHL. Furthermore, this is patient is 19 and unlikely to have head and neck cancer causing local lymphadenopathy and infection does not explain the weight loss over 3 months.

How well did you know this?
1
Not at all
2
3
4
5
Perfectly
28
Q

A 17 year old male presents with a 1 week history of fever, malaise, pain on swallowing & has found lumps in the neck. On examination a tender scrotal swelling is also noted.

Choose the most likely diagnosis from the list of options.
A.	Thyroid goitre
B.	Thyroiditis
C.	TB lymphadenitis
D.	Thyroglossal cyst
E.	Pharyngeal pouch
F.	Salivary gland tumour
G.	Lymphadenopathy secondary to infection
H.	Glandular fever
I.	Carotid body tumour
J.	Parotitis
K.	Mumps
L.	Lymphadenopathy secondary to malignancy
M.	Hodgkin's lymphoma
N.	Thyroid cancer
A

K. Mumps

Mumps is normally characterised by the hallmark finding of parotitis and swelling of the parotid gland(s) which is present in 95% of symptomatic mumps. Mumps epididymo-orchitis is also a common feature and is seen here as a tender scrotal swelling. The history this patient gives makes you think of an infection and the co-existent orchitis and neck lumps makes you think of mumps. Treatment of this viral infection involves isolation and supportive care with paracetamol or ibuprofen.

How well did you know this?
1
Not at all
2
3
4
5
Perfectly
29
Q

A 58 year old male presents to his GP with a lump in the neck and weight loss. He reports that he suffered from a mild upper respiratory infection in the previous week. On examination there is a hard mobile lump in the supraclavicular fossa.

Choose the most likely diagnosis from the list of options.
A.	Thyroid goitre
B.	Thyroiditis
C.	TB lymphadenitis
D.	Thyroglossal cyst
E.	Pharyngeal pouch
F.	Salivary gland tumour
G.	Lymphadenopathy secondary to infection
H.	Glandular fever
I.	Carotid body tumour
J.	Parotitis
K.	Mumps
L.	Lymphadenopathy secondary to malignancy
M.	Hodgkin's lymphoma
N.	Thyroid cancer
A

L. Lymphadenopathy secondary to malignancy

This 58 year old male reports weight loss. Combined with the presence of Virchow’s node (Troisier’s sign), a hard enlarged node in the left supraclavicular fossa, this points towards a malignancy in the abdominal cavity. The lymph drainage of the abdominal cavity drains into Virchow’s node as the lymph drains most of the body from the thoracic duct and enters the venous circulation at the left subclavian vein. An enlarged right supraclavicular LN may indicate thoracic malignancies such as lung cancer. The mild URTI is a red herring.

How well did you know this?
1
Not at all
2
3
4
5
Perfectly
30
Q

An 18 year old girl presents with a midline swelling in the neck which has recently become tender. It moves on swallowing & on protusion of the tongue.

Choose the most likely diagnosis from the list of options.
A.	Thyroid goitre
B.	Thyroiditis
C.	TB lymphadenitis
D.	Thyroglossal cyst
E.	Pharyngeal pouch
F.	Salivary gland tumour
G.	Lymphadenopathy secondary to infection
H.	Glandular fever
I.	Carotid body tumour
J.	Parotitis
K.	Mumps
L.	Lymphadenopathy secondary to malignancy
M.	Hodgkin's lymphoma
N.	Thyroid cancer
A

D. Thyroglossal cyst

This midline neck swelling moves up on both swallowing and tongue protrusion making this a thyroglossal cyst. It is a cyst that forms from a remnant thyroglossal duct and can hence develop anywhere along the length of this embryological duct, which is a midline structure between the foramen caecum at the back of the tongue and the thyroid gland.

How well did you know this?
1
Not at all
2
3
4
5
Perfectly
31
Q

A 28 year old woman has developed rapid weight loss & palpitations. You notice lid lag & a goitre on examination.

Choose the most likely diagnosis from the list of options.
A.	Streptococcus pyogenes
B.	Carotid artery aneurysm
C.	Heart failure
D.	Hypothyroidism
E.	Hodgkin's disease
F.	Grave's disease
G.	TB abscess
H.	Myasthenia gravis
I.	De Quervain's thyroiditis
J.	Superior vena cava syndrome
K.	Thyroglossal cyst
L.	Stomach cancer
M.	Pancreatic carcinoma
A

F. Grave’s disease

This woman has symptoms of hyperthyroidism (weight loss and palpitations) and a goitre. In countries where sufficient iodine intake is not an issue, Graves’ disease is the most common cause of hyperthyroidism. Graves’ gives a diffuse goitre. Peripheral manifestations such as ophthalmopathy, pretibial myxoedema and hyperthyroid acropachy do not occur with other causes of hyperthyroidism. Ophthalmopathy includes lid retraction, exophthalmos and eye movement restriction leading to diplopia. Acropachy is an uncommon manifestation presenting as clubbing with soft tissue swelling. Pretibial myxoedema is almost always associated with ophthalmopathy. Treatment aims to normalise thyroid function and is achieved by radioactive iodine, antithyroid medications or with surgery. They are all effective and relatively safe options. Symptomatic therapy is given with beta blockers such as propranolol.

How well did you know this?
1
Not at all
2
3
4
5
Perfectly
32
Q

A 17 year old man has noticed a painless smooth swelling just above the suprasternal notch. He has had the swelling for 2 years & is well. It moves with swallowing & tongue protusion.

Choose the most likely diagnosis from the list of options.
A.	Streptococcus pyogenes
B.	Carotid artery aneurysm
C.	Heart failure
D.	Hypothyroidism
E.	Hodgkin's disease
F.	Grave's disease
G.	TB abscess
H.	Myasthenia gravis
I.	De Quervain's thyroiditis
J.	Superior vena cava syndrome
K.	Thyroglossal cyst
L.	Stomach cancer
M.	Pancreatic carcinoma
A

K. Thyroglossal cyst

This midline neck swelling moves up on both swallowing and tongue protrusion making this a thyroglossal cyst. It is a cyst that forms from a remnant thyroglossal duct and can hence develop anywhere along the length of this embryological duct, which is a midline structure between the foramen caecum at the back of the tongue and the thyroid gland.

How well did you know this?
1
Not at all
2
3
4
5
Perfectly
33
Q

A 17 year old girl has developed an acutely painful goitre. She has a fever & pain on swallowing.

Choose the most likely diagnosis from the list of options.
A.	Streptococcus pyogenes
B.	Carotid artery aneurysm
C.	Heart failure
D.	Hypothyroidism
E.	Hodgkin's disease
F.	Grave's disease
G.	TB abscess
H.	Myasthenia gravis
I.	De Quervain's thyroiditis
J.	Superior vena cava syndrome
K.	Thyroglossal cyst
L.	Stomach cancer
M.	Pancreatic carcinoma
A

I. De Quervain’s thyroiditis

This girl has de Quervain’s thyroiditis (which can also be called subacute granulomatous thyroiditis) which is inflammation of the thyroid characterised by a triphasic course where there is transient thyrotoxicosis followed by a hypothyroid phase before a return to euthyroidism. The thyrotoxic phase (symptoms of hyperthyroidism may be present) is characterised by pain and tenderness of the thyroid, which tends to be larger, firm and tender to touch. This girl also has a fever and neck pain which is making it painful for her to swallow. This is a self-limiting condition and no specific treatment is needed though NSAIDs and beta blockers can be used for symptomatic relief. Roughly 30-40% describe a prior viral infection.

How well did you know this?
1
Not at all
2
3
4
5
Perfectly
34
Q

A 22 year old man has developed multiple hard swellings on the left side of the neck. He has had night sweats & anorexia for 2 months.

Choose the most likely diagnosis from the list of options.
A.	Streptococcus pyogenes
B.	Carotid artery aneurysm
C.	Heart failure
D.	Hypothyroidism
E.	Hodgkin's disease
F.	Grave's disease
G.	TB abscess
H.	Myasthenia gravis
I.	De Quervain's thyroiditis
J.	Superior vena cava syndrome
K.	Thyroglossal cyst
L.	Stomach cancer
M.	Pancreatic carcinoma
A

E. Hodgkin’s disease

This is a case of lymphoma. Reed-Sternberg cells are binucleate cells characteristically seen in Hodgkin’s lymphoma. Hodgkin’s is localised to a single group of nodes (normally the cervical and/or supraclavicular) and extranodal involvement is rare. Mediastinal involvement is common. Spread is contiguous and B symptoms may be present such as a low grade fever, weight loss and night sweats. Pruritis may be found in approximately 10% of cases but has no prognostic significance. 50% of cases is associated with EBV infection and distribution is bimodal with peaks in young and old. There is classically pain in lymph nodes on alcohol consumption.

How well did you know this?
1
Not at all
2
3
4
5
Perfectly
35
Q

A 67 year old male smoker has developed gross oedema of the neck & face. You also notice swelling of both hands.

Choose the most likely diagnosis from the list of options.
A.	Streptococcus pyogenes
B.	Carotid artery aneurysm
C.	Heart failure
D.	Hypothyroidism
E.	Hodgkin's disease
F.	Grave's disease
G.	TB abscess
H.	Myasthenia gravis
I.	De Quervain's thyroiditis
J.	Superior vena cava syndrome
K.	Thyroglossal cyst
L.	Stomach cancer
M.	Pancreatic carcinoma
A

J. Superior vena cava syndrome

SVC syndrome occurs due to SVCO. The most common cause of this is malignancy (this smoker probably has lung cancer which is the most likely cause in those >50). There are also benign causes such as iatrogenically due to pacemaker leads and central venous catheters causing SVC thrombosis. Common symptoms are facial swelling and arm swelling as seen, as well as dyspnoea, cough and facial plethora. There may also be a headache, chest pain, blurry vision and stridor. These symptoms tend to be worse on bending forwards or lying down. The most useful imaging test is a chest CT with IV contrast which establishes the diagnosis and also shows the exact location of the pathology. A CXR in SVCO can show a widened mediastinum or the lung cancer which is the cause.

How well did you know this?
1
Not at all
2
3
4
5
Perfectly
36
Q

A 20 year old medical student is concerned about recent weight gain & lethargy & constipation. You notice bradycardia & a goitre. Her ankle reflexes are sluggish. Her periods have stopped.

Choose the most likely diagnosis from the list of options.
A.	Streptococcus pyogenes
B.	Carotid artery aneurysm
C.	Heart failure
D.	Hypothyroidism
E.	Hodgkin's disease
F.	Grave's disease
G.	TB abscess
H.	Myasthenia gravis
I.	De Quervain's thyroiditis
J.	Superior vena cava syndrome
K.	Thyroglossal cyst
L.	Stomach cancer
M.	Pancreatic carcinoma
A

D. Hypothyroidism

This medical student has hypothyroidism. Worldwide, the most common cause is iodine deficiency. Other causes include Hashimoto’s or secondary and tertiary hypothyroidism. It can also result from viral de Quervain’s thyroiditis or postpartum thyroiditis. Symptoms include those mentioned (weight gain, lethargy, sluggish reflexes, bradycardia and constipation) as well as depression, fatigue, constipation, cold intolerance, menstrual problems in females, dry skin and muscle cramps. Diagnosis is based on measurement of TSH and thyroid hormones. Treatment is by replacement of T4 with or without T3 in combination. If the patient has normal T3 and T4 but mildly elevated TSH, this is described as subclinical hypothyroidism.

How well did you know this?
1
Not at all
2
3
4
5
Perfectly
37
Q

A 72 year old man with weight loss has developed a hard swelling in the left supraclavicular fossa.

Choose the most likely diagnosis from the list of options.
A.	Streptococcus pyogenes
B.	Carotid artery aneurysm
C.	Heart failure
D.	Hypothyroidism
E.	Hodgkin's disease
F.	Grave's disease
G.	TB abscess
H.	Myasthenia gravis
I.	De Quervain's thyroiditis
J.	Superior vena cava syndrome
K.	Thyroglossal cyst
L.	Stomach cancer
M.	Pancreatic carcinoma
A

l. Stomach cancer

The presence of Virchow’s node (Troisier’s sign), a hard enlarged node in the left supraclavicular fossa, points towards a malignancy in the abdominal cavity. This is most often stomach cancer. The lymph drainage of the abdominal cavity drains into Virchow’s node as the lymph drains most of the body from the thoracic duct and enters the venous circulation at the left subclavian vein.

How well did you know this?
1
Not at all
2
3
4
5
Perfectly
38
Q

12 yr old boy presents with 4 weeks weight loss, polyuria and polydipsia.

Choose the most likely investigation from the list.
A.	ACTH (synacthen) stimulation test
B.	Dexamethasone suppression test
C.	Liver function tests
D.	Abdominal ultrasound
E.	Fasting blood glucose
F.	Full blood count
G.	Water deprivation test
H.	Dietary history
I.	T3, T4 and TSH levels 
J.	Urinary ketones
K.	Serum creatinine
L.	Serum aldosterone
M.	HbA1c levels
A

E. Fasting blood glucose

For the diagnosis of DM, symptomatic patients need a single random blood glucose of >11.1 or single fasting glucose of >7. Asymptomatic patients need two separate elevated readings for a diagnosis. Alternatively if there are borderline results, an OGTT can be conducted to see if plasma glucose is raised >11.1 two hours after an oral glucose load of 75g. A patient is said to have impaired fasting glucose if fasting glucose falls between 6.1-6.9. Impaired glucose tolerance is present if plasma glucose 2 hours after oral glucose load in OGTT falls between 7.8-11.0. This patient most likely has new onset T1DM.

How well did you know this?
1
Not at all
2
3
4
5
Perfectly
39
Q

44 yr old woman presents with tachycardia, atrial fibrillation, double vision and swelling above her ankles. She has lid lag on examination.

Choose the most likely investigation from the list.
A.	ACTH (synacthen) stimulation test
B.	Dexamethasone suppression test
C.	Liver function tests
D.	Abdominal ultrasound
E.	Fasting blood glucose
F.	Full blood count
G.	Water deprivation test
H.	Dietary history
I.	T3, T4 and TSH levels 
J.	Urinary ketones
K.	Serum creatinine
L.	Serum aldosterone
M.	HbA1c levels
A

I. T3, T4 and TSH levels

This patient has Graves’ disease. T3, T4 and TSH need to be measured. We would expect to find a suppressed TSH with elevated T3 and T4 (unless in T3 toxicosis).

How well did you know this?
1
Not at all
2
3
4
5
Perfectly
40
Q

42 yr old man has hypertension, hyperglycaemia, myopathy, thinning of skin, buffalo hump and truncal obesity.

Choose the most likely investigation from the list.
A.	ACTH (synacthen) stimulation test
B.	Dexamethasone suppression test
C.	Liver function tests
D.	Abdominal ultrasound
E.	Fasting blood glucose
F.	Full blood count
G.	Water deprivation test
H.	Dietary history
I.	T3, T4 and TSH levels 
J.	Urinary ketones
K.	Serum creatinine
L.	Serum aldosterone
M.	HbA1c levels
A

B. Dexamethasone suppression test

This patient has Cushing’s and will need a dexamethasone suppression test which is first line in anyone with suspected Cushing’s (unless they are on medications which will affect the result of the test), though alternatively a 24 hour urinary free coritsol and late night salivary cortisol can be done. The dexamethasone suppression test takes the form of either an overnight 1mg or a 48 hour 2mg test. Unsuppressed cortisol indicates Cushing’s syndrome.

How well did you know this?
1
Not at all
2
3
4
5
Perfectly
41
Q

18 yr old girl complains of her appearance. She is much too fat according to her. She also missed periods and has become hairier. On examination she is 10kg overweight.

Choose the most likely investigation from the list.
A.	ACTH (synacthen) stimulation test
B.	Dexamethasone suppression test
C.	Liver function tests
D.	Abdominal ultrasound
E.	Fasting blood glucose
F.	Full blood count
G.	Water deprivation test
H.	Dietary history
I.	T3, T4 and TSH levels 
J.	Urinary ketones
K.	Serum creatinine
L.	Serum aldosterone
M.	HbA1c levels
A

D. Abdominal ultrasound

This patient’s history makes you think PCOS. Hirsutism, irregular and infrequent periods and weight gain are all features, as are acne, scalp hair loss and infertility. Hypertension is sometimes associated with this syndrome. On examination, sweaty/oily skin may be found and acanthosis nigricans may also be seen. An ultrasound is by no means the first test to order, as 1/4 of normal women and women with other problems like hyperprolactinaemia may also have polycystic ovaries, and they are present in 3/4 of those with PCOS. Serum androgens can be measured or the diagnosis can be made clinically.

How well did you know this?
1
Not at all
2
3
4
5
Perfectly
42
Q

34 yr old old man with insidious onset weakness and weight loss. On examination he has hyperpigmentation of the palmar creases and postural hypotension.

Choose the most likely investigation from the list.
A.	ACTH (synacthen) stimulation test
B.	Dexamethasone suppression test
C.	Liver function tests
D.	Abdominal ultrasound
E.	Fasting blood glucose
F.	Full blood count
G.	Water deprivation test
H.	Dietary history
I.	T3, T4 and TSH levels 
J.	Urinary ketones
K.	Serum creatinine
L.	Serum aldosterone
M.	HbA1c levels
A

A. ACTH (synacthen) stimulation test

Hyperpigmentation in the palmar creases points towards Addison’s disease. Hyperpigmentation due to excess ACTH production can be mucosal or cutaneous and is more pronounced in the palms, knuckles and around scars. MSH is a byproduct of the production of ACTH from the cleavage of POMC. Sodium is low and potassium elevated. Vomiting is present in 75% of patients and nausea is a common finding. Additionally, postural hypotension may be present. The presence of other autoimmune diseases is a risk factor for the development of Addison’s. Diagnosis of Addison’s can be made on an ACTH stimulation test (synacthen test) whereby serum cortisol remains low despite the administration of synthetic ACTH. In an emergency, treatment should not be delayed by diagnostic testing.

How well did you know this?
1
Not at all
2
3
4
5
Perfectly
43
Q

64 yr old woman complaining of severe back pain for some weeks. For the last few days she has been very constipated and has been vomiting for 24 hours. She has been a smoker for many years and has had 3 courses of antibiotics for chest infections over the last 3 months.

For each patient choose a the most likely diagnosis from the list of options.
A.	Hyponatraemia
B.	Hypernatraemia
C.	Hypercalcaemia
D.	Hypothyroidism
E.	Hypercapnia
F.	Addisons disease
G.	Metabolic acidosis
H.	Hypoglycaemia
I.	Hypermagnesaemia
J.	Hypokalaemia
K.	Hyperkalaemia
L.	Hypocalcaemia
A

C. Hypercalcaemia

90% of hypercalcaemia is caused by primary hyperparathyroidism or cancer. Cancer is the likely cause in this woman. Malignancy can cause hypercalcaemia either by direct bony involvement leading to osteolytic lesions or paraneoplastic syndromes involving PTHrp release. The tumour is typically very advanced if hypercalcaemia is a feature. Less common causes include vitamin D overdose, hyperthyroidism, immobilisation, Paget’s and milk-alkali syndrome. The classic bone disease in hyperparathyroidism is osteitis fibrosa cystica which causes pain. Symptoms of high calcium include confusion, constipation, polyuria, polydipsia, depression, kidney stones and lethargy. This can be remembered by ‘stones, bones, abdominal groans and psychiatric moans’. The serum PTH level is elevated in primary hyperparathyroidism whereas it may be very low in malignancy due to negative feedback.

How well did you know this?
1
Not at all
2
3
4
5
Perfectly
44
Q

21 yr old woman has been complaining of increasing tiredness and feeling thirsty. Over the last day she had been confused. On arrival in A&E she was noted to have deep sighing respiration.

For each patient choose a the most likely diagnosis from the list of options.
A.	Hyponatraemia
B.	Hypernatraemia
C.	Hypercalcaemia
D.	Hypothyroidism
E.	Hypercapnia
F.	Addisons disease
G.	Metabolic acidosis
H.	Hypoglycaemia
I.	Hypermagnesaemia
J.	Hypokalaemia
K.	Hyperkalaemia
L.	Hypocalcaemia
A

G. Metabolic acidosis

90% of hypercalcaemia is caused by primary hyperparathyroidism or cancer. Cancer is the likely cause in this woman. Malignancy can cause hypercalcaemia either by direct bony involvement leading to osteolytic lesions or paraneoplastic syndromes involving PTHrp release. The tumour is typically very advanced if hypercalcaemia is a feature. Less common causes include vitamin D overdose, hyperthyroidism, immobilisation, Paget’s and milk-alkali syndrome. The classic bone disease in hyperparathyroidism is osteitis fibrosa cystica which causes pain. Symptoms of high calcium include confusion, constipation, polyuria, polydipsia, depression, kidney stones and lethargy. This can be remembered by ‘stones, bones, abdominal groans and psychiatric moans’. The serum PTH level is elevated in primary hyperparathyroidism whereas it may be very low in malignancy due to negative feedback.

How well did you know this?
1
Not at all
2
3
4
5
Perfectly
45
Q

56 yr old man who had been on amiodarone for 2 years for atrial fibrillation complained of increasing tiredness. He had gained 5kg in weight over the last 6 months. His wife complained that he felt cold even in a warm room.

For each patient choose a the most likely diagnosis from the list of options.
A.	Hyponatraemia
B.	Hypernatraemia
C.	Hypercalcaemia
D.	Hypothyroidism
E.	Hypercapnia
F.	Addisons disease
G.	Metabolic acidosis
H.	Hypoglycaemia
I.	Hypermagnesaemia
J.	Hypokalaemia
K.	Hyperkalaemia
L.	Hypocalcaemia
A

D. Hypothyroidism

Amiodarone can cause both hyperthyroidism (Jod-Basedow effect) and hypothyroidism (Wolff-Chaikoff effect). Amiodarone is 37.3% iodine by weight and is structurally similar to thyroxine.

How well did you know this?
1
Not at all
2
3
4
5
Perfectly
46
Q

22 yr old woman was admitted with abdominal pain. Her mother had pernicious anaemia. On examination vitiligo was noted and the creases of her palms appeared dark. BP was 100/50 lying and 80/60 standing.

For each patient choose a the most likely diagnosis from the list of options.
A.	Hyponatraemia
B.	Hypernatraemia
C.	Hypercalcaemia
D.	Hypothyroidism
E.	Hypercapnia
F.	Addisons disease
G.	Metabolic acidosis
H.	Hypoglycaemia
I.	Hypermagnesaemia
J.	Hypokalaemia
K.	Hyperkalaemia
L.	Hypocalcaemia
A

F. Addison’s disease

The presence of other autoimmune diseases such as T1DM, RA, vitiligo are all risk factors to the development of Addison’s. There is also a FH of pernicious anaemia. Postural hypotension and hyperpigmentation in the palmar creases all point towards Addison’s disease. Hyperpigmentation due to excess ACTH production can be mucosal or cutaneous and is more pronounced in the palms, knuckles and around scars. MSH is a byproduct of the production of ACTH from the cleavage of POMC. If serum electrolytes were done, sodium would be low and potassium elevated.

How well did you know this?
1
Not at all
2
3
4
5
Perfectly
47
Q

43 yr old woman, 24 hours after a total thyroidectomy for Grave’s disease develops stridor and carpo-pedal spasm.

For each patient choose a the most likely diagnosis from the list of options.
A.	Hyponatraemia
B.	Hypernatraemia
C.	Hypercalcaemia
D.	Hypothyroidism
E.	Hypercapnia
F.	Addisons disease
G.	Metabolic acidosis
H.	Hypoglycaemia
I.	Hypermagnesaemia
J.	Hypokalaemia
K.	Hyperkalaemia
L.	Hypocalcaemia
A

L. Hypocalcaemia

Hypocalcaemia can develop as a complication of thyroid surgery due to the loss of parathyroid glands which produce PTH. Trousseau’s sign is carpal spasm when a blood pressure cuff is used for several minutes. Carpopedal spasm that occurs with hypocalcaemia is a painful spasm and could be the presenting sign. Chvostek’s sign is twitching of the perioral muscles in response to tapping over the facial nerve at the ear. If urgent replacement is necessary, calcium gluconate can be given IV. It is preferred over calcium chloride as it causes less tissue necrosis if it leaks out. It is worth noting that digoxin may be ineffective until serum calcium is restored to normal.

How well did you know this?
1
Not at all
2
3
4
5
Perfectly
48
Q

60 year old T2DM presents with gradually worsening diabetes despite maximum oral medication. He has ketones in his urine and is unwell.

Match the appropriate treatment to these scenarios.
A.	Insulin
B.	Gliclazide
C.	Acarbose (Alpha glucosidase inhibitor)
D.	Diet and exercise
E.	Glucagon
F.	Diet and exercise
G.	Insulin
H.	Pioglitazone (Thiazolidinedione)
I.	Repaglinide (Meglitinide)
J.	Metformin
A

G. Insulin

Insulin therapy is recommended in T2DM not controlled on 2 oral agents and is usually started with basal insulin at night time (such as glargine and detemir, your long acting insulins). In any case, this patient is getting worse despite maximum oral medication, which may have included even a third non-insulin agent such as a DPP-4 inhibitor. Many patients will also need rapid acting insulin (such as lispro and aspart) before each meal. There are also other options for basal-bolus regimes such as intermediate acting insulin (NPH) and short-acting insulins. Metformin can be continued but sulphonylurea should be tapered as insulin doses increase. Flexible dosing is the preferred regime but pre-mixed insulin is an option. Insulin can also be used at the time for diagnosis in cases of marked hyperglycaemia. Self-monitored blood glucose testing will be necessary too and side effects include weight gain and hypoglycaemia.

This question requires you to have some knowledge of the stratified glycaemic management for DM. The general goal is HbA1c <7% (from 2011, these values are expressed in mmol/mol, so this would be <53 mmol/mol). HbA1c is glycated Hb and provides an estimation of glycaemic control over the life span of red blood cells (around 60 days).

BP control with an ACE inhibitor, lipid control with a statin, lifestyle changes and stopping smoking are all crucial in the treatment of T2DM and to try to reduce the mortality associated with macrovascular complications. First line intervention in newly diagnosed diabetes is diet and lifestyle advice and changes. If this fails to keep blood sugars in check then the patient will need to be started on an oral hypoglycaemic, and the first line is metformin. Metformin is a biguanide and suppresses hepatic glucose production.

How well did you know this?
1
Not at all
2
3
4
5
Perfectly
49
Q

An overweight 45 year old man who was diagnosed with diabetes a few months ago still has fasting glucose of 11 and HbA1C of 8.8%

Match the appropriate treatment to these scenarios.
A.	Insulin
B.	Gliclazide
C.	Acarbose (Alpha glucosidase inhibitor)
D.	Diet and exercise
E.	Glucagon
F.	Diet and exercise
G.	Insulin
H.	Pioglitazone (Thiazolidinedione)
I.	Repaglinide (Meglitinide)
J.	Metformin
A

J. Metformin

It has been a few months already, and we can assume that diet and lifestyle changes have not had an effect. This patient will need to be started on metformin.

How well did you know this?
1
Not at all
2
3
4
5
Perfectly
50
Q

56 yr old man with diabetes who is on maximum therapy of gliclazide needs higher control but is unable to tolerate metformin or acarbose and refuses any injections

Match the appropriate treatment to these scenarios.
A.	Insulin
B.	Gliclazide
C.	Acarbose (Alpha glucosidase inhibitor)
D.	Diet and exercise
E.	Glucagon
F.	Diet and exercise
G.	Insulin
H.	Pioglitazone (Thiazolidinedione)
I.	Repaglinide (Meglitinide)
J.	Metformin
A

H. Pioglitazone (Thiazolidinedione)

This man is on a sulphonylurea which is an insulin secretagogue. It is an old drug with a long track record and is being used in this case as the patient does not tolerate metformin, which would otherwise have been first line. Sulphonylureas can cause hypoglycaemia. They act at the pancreas by blocking ATP sensitive potassium channels which leads to beta cell depolarisation and calcium influx, which causes increased insulin release. They also act on peripheral insulin receptors and have hepatic effects. This man is on maximum therapy of gliclazide and is refusing any injections so this rules out insulin therapy so we must consider non-insulin agents as add-ons. These include alpha glucosidase inhibitors (but our patient also cannot tolerate acarbose), DPP-4 inhibitors (‘gliptins’), GLP1 agonists (exenatide) or TZDs. The latter is the only available option on the list.

How well did you know this?
1
Not at all
2
3
4
5
Perfectly
51
Q
15 yr old diabetic is shaky, sweaty, trembling and pale. He then falls. Bmstix = 2mmol/l
Match the appropriate treatment to these scenarios.
A.	Insulin
B.	Gliclazide
C.	Acarbose (Alpha glucosidase inhibitor)
D.	Diet and exercise
E.	Glucagon
F.	Diet and exercise
G.	Insulin
H.	Pioglitazone (Thiazolidinedione)
I.	Repaglinide (Meglitinide)
J.	Metformin
A

E. Glucagon

This patient is suffering from hypoglycaemia present when glucose drops <3mmol/L. Symptoms include sweating, weakness, drowsiness, palpitations and anxiety. This patient has collapsed and is presumably not able to take oral glucose or sugar (which in any case is not an option on the list). Patients with either long standing DM or on beta blockers may become unaware of hypoglycaemia and become profoundly hypoglycaemic before symptoms develop. In DM, hypoglycaemia is usually secondary to insulin or oral hypoglycaemics. IM glucagon can be given instead in this case.

52
Q

26 yr old pregnant lady at the end of her pregnancy develops gestational diabetes.

Match the appropriate treatment to these scenarios.
A.	Insulin
B.	Gliclazide
C.	Acarbose (Alpha glucosidase inhibitor)
D.	Diet and exercise
E.	Glucagon
F.	Diet and exercise
G.	Insulin
H.	Pioglitazone (Thiazolidinedione)
I.	Repaglinide (Meglitinide)
J.	Metformin
A

F. Diet and exercise

Diabetes occuring during pregnancy are mostly adequately treated on diet alone. Just reduce carbohydrate intake and monitor glucose levels. If uncontrolled or markedly severe, insulin therapy can be used. It is however worth noting that after delivery of the placenta, there is a big reduction in insulin requirement which may be factored in. Initial postpartum insulin needs are lower than pre-pregnancy. Treatment aims to lower glucose levels to avoid fetal macrosomia and complications. Subsequent long term risk for T2DM is high.

53
Q

23 yr old man who is fit and well has a checkup and is found to have glycosuria. Fasting glucose is 8.4mmol/l.HbA1C is 6.5%

Match the appropriate treatment to these scenarios.
A.	Insulin
B.	Gliclazide
C.	Acarbose (Alpha glucosidase inhibitor)
D.	Diet and exercise
E.	Glucagon
F.	Diet and exercise
G.	Insulin
H.	Pioglitazone (Thiazolidinedione)
I.	Repaglinide (Meglitinide)
J.	Metformin
A

D. Diet and exercise

This patient is not symptomatic and so needs further tests before a diagnosis can be made. However, regardless, first line intervention is diet and lifestyle changes in newly diagnosed DM.

54
Q

26 yr old alcoholic diabetic is found in a coma and taken to hospital. Blood glucose is 28mmmol/l

Match the appropriate treatment to these scenarios.
A.	Insulin
B.	Gliclazide
C.	Acarbose (Alpha glucosidase inhibitor)
D.	Diet and exercise
E.	Glucagon
F.	Diet and exercise
G.	Insulin
H.	Pioglitazone (Thiazolidinedione)
I.	Repaglinide (Meglitinide)
J.	Metformin
A

A. Insulin

This is DKA (HHS will have a higher blood glucose and tends not to present in a coma). If IV saline is an option in EMQs, it is a better option than insulin, which is also needed, but the main thing that needs to be corrected is the severe intravascular volume depletion and to restore tissue perfusion. When glucose reaches 11.1mmol, fluid should be changed to 5% dextrose to prevent hypoglycaemia. Insulin should be held until potassium is at least 3.3 mmol/L (remember insulin moves potassium into cells) and a continuous infusion is recommended (with new DKA treatment guidelines, the ‘sliding scale’ is now a thing of the past). If interested, look up the latest DKA treatment guidelines for more information.

55
Q

A 35 yr old woman complains of weight loss, tremors, palpitations, large appetite and diarrhoea. Examination revealed tachycardia over 120beats/min and exopthalmos. She appeared anxious.

Choose the most likely diagnosis from the list of options.
A.	Myxoedema
B.	Viral thyroiditis
C.	Anaplastic carcinoma
D.	Toxic multinodular goitre
E.	Thyroglossal cyst
F.	Grave's disease
G.	Medullary thyroid carcinoma
H.	Carotid body tumour
I.	Pituitary failure
J.	Infected dermoid cyst
K.	Papillary thyroid carcinoma
L.	Anxiety state
A

F. Grave’s disease

This patient is obviously hyperthyroid with the weight loss despite an increased appetite, tremor, palpitations, tachycardia and diarrhoea. This patient has Graves’ disease, which is the most common cause of hyperthyroidism in countries with an adequate iodine intake. Peripheral manifestations, such as in this case exopthalmos (and pretibial myxoedema, hyperthyroid acropachy), do not occur with other causes of hyperthyroidism. Where there is pretibial myxoedema, there is almost always opthalmopathy too. Treatment aims to normalise thyroid function and is achieved by radioactive iodine, antithyroid medications or with surgery. They are all effective and relatively safe options. Symptomatic therapy is given with beta blockers such as propranolol.

56
Q

50 yr old woman complains of weight loss, tremor of hands, palpitations, large appetite, diarrhoea and an irregular in the neck. The patient is tachycardiac and eye examination revealed no exopthalmos or lid lag.

Choose the most likely diagnosis from the list of options.
A.	Myxoedema
B.	Viral thyroiditis
C.	Anaplastic carcinoma
D.	Toxic multinodular goitre
E.	Thyroglossal cyst
F.	Grave's disease
G.	Medullary thyroid carcinoma
H.	Carotid body tumour
I.	Pituitary failure
J.	Infected dermoid cyst
K.	Papillary thyroid carcinoma
L.	Anxiety state
A

D. Toxic multinodular goitre

This patient is again obviously hyperthyroid with the symptoms she is presenting with but does not have the peripheral stigmata of Graves’. This is a case of toxic multinodular goitre characterised by the irregular goitre, rather than the smooth goitre of Graves’. There may be substernal extension. It is most common in older patients and is associated with head and neck irradiation and iodine deficiency. TSH is the initial screening test and if supressed, T4/T3 levels are measured. A thyroid scan and uptake will show multiple hot and cold areas consistent with areas of autonomy and areas of suppression. Definitive treatment is commonly given in the form of radioactive iodine.

57
Q

24 yr old woman complains of weight loss, tremor of hands, palpitations and loss of appetite. Examination revealed tachycardia and she appeared nervous. Investigation revealed TSH 1.0 (range 0.3-4.5) and ESR within normal limits

Choose the most likely diagnosis from the list of options.
A.	Myxoedema
B.	Viral thyroiditis
C.	Anaplastic carcinoma
D.	Toxic multinodular goitre
E.	Thyroglossal cyst
F.	Grave's disease
G.	Medullary thyroid carcinoma
H.	Carotid body tumour
I.	Pituitary failure
J.	Infected dermoid cyst
K.	Papillary thyroid carcinoma
L.	Anxiety state
A

L. Anxiety state

TSH is normal so there is no thyroid disorder here despite the patient’s symptoms and the patient also appears nervous. Anxiety would explain the patient’s symptoms here and weight loss may be incorporated as part of an anxiety disorder.

58
Q

24 yr old woman complains of weight loss, tremor of hands, palpitations and pain in the neck radiating to the left ear and tiredness. Examination revealed tender swelling in the neck, tachycardia and a fever. TSH <0.05 (range 0.3-4.5) and ESR 60mm/hr

Choose the most likely diagnosis from the list of options.
A.	Myxoedema
B.	Viral thyroiditis
C.	Anaplastic carcinoma
D.	Toxic multinodular goitre
E.	Thyroglossal cyst
F.	Grave's disease
G.	Medullary thyroid carcinoma
H.	Carotid body tumour
I.	Pituitary failure
J.	Infected dermoid cyst
K.	Papillary thyroid carcinoma
L.	Anxiety state
A

B. Viral thyroiditis

This woman has de Quervain’s (viral) thyroiditis (which can also be called subacute granulomatous thyroiditis) which is inflammation of the thyroid characterised by a triphasic course where there is transient thyrotoxicosis followed by a hypothyroid phase before a return to euthyroidism. The thyrotoxic phase (symptoms of hyperthyroidism may are present here) is characterised by pain and tenderness of the thyroid, which tends to be larger, firm and tender to touch. This woman also has a fever and neck pain. This is a self-limiting condition and no specific treatment is needed though NSAIDs and beta blockers can be used for symptomatic relief. Roughly 30-40% describe a prior viral infection.

59
Q

An 18 yr old woman complains of a swelling in the neck. Examination revealed the swelling is painless, non tender. There is no weight loss, no tremor of hands or palpitations. Swelling moves up and down with protrusion and intrusion of tongue respectively. Investigation revealed normal TSH and ESR.

Choose the most likely diagnosis from the list of options.
A.	Myxoedema
B.	Viral thyroiditis
C.	Anaplastic carcinoma
D.	Toxic multinodular goitre
E.	Thyroglossal cyst
F.	Grave's disease
G.	Medullary thyroid carcinoma
H.	Carotid body tumour
I.	Pituitary failure
J.	Infected dermoid cyst
K.	Papillary thyroid carcinoma
L.	Anxiety state
A

E. Thyroglossal cyst

This midline neck swelling moves up on tongue protrusion (and swallowing) making this a thyroglossal cyst. It is a cyst that forms from a remnant thyroglossal duct and can hence develop anywhere along the length of this embryological duct, which is a midline structure between the foramen caecum at the back of the tongue and the thyroid gland.

60
Q

A 50 yr old man complains of a lump in neck on the left side. Examination revealed painless, non tender lump on the left side of the neck which moves on swallowing. Investigation revealed normal TSH and ESR. Fine needle aspiration revealed ‘Orphan Annie’ eyes and psammoma bodies.

Choose the most likely diagnosis from the list of options.
A.	Myxoedema
B.	Viral thyroiditis
C.	Anaplastic carcinoma
D.	Toxic multinodular goitre
E.	Thyroglossal cyst
F.	Grave's disease
G.	Medullary thyroid carcinoma
H.	Carotid body tumour
I.	Pituitary failure
J.	Infected dermoid cyst
K.	Papillary thyroid carcinoma
L.	Anxiety state
A

K. Papillary thyroid carcinoma

TSH is normal and the examination findings point to a carcinoma, which most commonly presents like this – an asymptomatic nodule. If TSH were suppressed then this would suggest hyperthyroidism, or a hot nodule, in which case the incidence of cancer is very low. If TSH, like this case, is normal, then a FNA is indicated and cytology may then help to tell cancer type. Histology cannot distinguish follicular adenomas and carcinomas but is used for diagnosis of follicular carcinoma (when combined with other features) instead of cytology. There are 5 types of thyroid cancer: papillary (most common), follicular, medullary (about a quarter are familial e.g. MEN), anaplastic (worst prognosis) and lymphoma. Orphan Annie eyes and psammoma bodies are seen in papillary cancer. Average 10 year survival is >90%. This is a hard question.

61
Q

60 yr old woman complains of weight gain, tiredness, loss of appetite. Examination revealed bradycardia. Investigations revealed TSH 30 (range 0.3-4.5). ECG showed ischaemic changes.

Choose the most likely diagnosis from the list of options.
A.	Myxoedema
B.	Viral thyroiditis
C.	Anaplastic carcinoma
D.	Toxic multinodular goitre
E.	Thyroglossal cyst
F.	Grave's disease
G.	Medullary thyroid carcinoma
H.	Carotid body tumour
I.	Pituitary failure
J.	Infected dermoid cyst
K.	Papillary thyroid carcinoma
L.	Anxiety state
A

A. Myxoedema

This patient has hypothyroidism (myxoedma). Worldwide, the most common cause is iodine deficiency. Other causes include Hashimoto’s or secondary and tertiary hypothyroidism. It can also result from viral de Quervain’s thyroiditis or postpartum thyroiditis. Symptoms include those mentioned (weight gain, fatigue, loss of appetite and bradycardia) as well as depression, sluggish reflexes, constipation, cold intolerance, menstrual problems in females, dry skin and muscle cramps. Diagnosis is based on measurement of TSH and thyroid hormones. Treatment is by replacement of T4 with or without T3 in combination. If the patient has normal T3 and T4 but mildly elevated TSH, this is described as subclinical hypothyroidism.

62
Q

Mrs. Patel aged 28 is an immigrant from India. She is a vegetarian. Examination is mostly unremarkable, but she looks rather pale & has spoon shaped fingernails.

All the following patients complain of feeling tired all the time. In addition, they have the following features. Select the most likely diagnosis from the list of options.
A.	RA
B.	Sleep apnoea syndrome
C.	Carcinoma of caecum
D.	AIDS
E.	IDA
F.	Hypothyroidism
G.	Pernicious anaemia
H.	Chronic fatigue syndrome
I.	MS
J.	Depression
A

E. IDA

This woman has IDA which is a microcytic hypochromic anaemia characterised by low serum iron, high TIBC and low transferrin saturation and serum ferritin. There is spooning of the nails (koilonychia) which is a sign. Nail chances begin with thinning and then flattening before spooning occurs. Pallor is also a sign of anaemia. Patients can often complain of faitgue, cravings, hair loss and there may also be glossitis and angular stomatitis. Initial treatment is with oral iron. The cause here is most likely dietary where intake does not cover losses. This can be attributed to the patient’s vegetarian diet. It is worth noting that iron absorption is enhanced by vitamin C whereas it is inhibited by tea, wine and calcium. However, it is worth noting that other causes of IDA include loss bleeding, which is the principal cause, and further investigations should be done if a cause is not obviously present. IDA is not an end diagnosis and has many causes. Worldwide, the most common cause is hookworm infection, and in the UK, menstrual losses in women. Causes of IDA can be broadly divided into 4 categories: decreased intake, increased loss, increased requirements (such as in pregnancy), and unknown.

63
Q

Mrs. Brown, aged 25, complains of poor concentration & memory, being irritable & unable to cope with her 4 month old baby & all the domestic chores. She wakes up early & feels worse first thing in the morning.

All the following patients complain of feeling tired all the time. In addition, they have the following features. Select the most likely diagnosis from the list of options.
A.	RA
B.	Sleep apnoea syndrome
C.	Carcinoma of caecum
D.	AIDS
E.	IDA
F.	Hypothyroidism
G.	Pernicious anaemia
H.	Chronic fatigue syndrome
I.	MS
J.	Depression
A

J. Depression

Chronic fatigue syndrome can only be diagnosed if there is no other explanation for the patient’s impairing fatigue, which is persistent and lasts >6 months. This case is depression, not CFS (the patient doesn’t even complain of feeling tired in this case). This woman has many risk factors: recent childbirth, stress, female sex. While she doesn’t directly complain of a depressed mood or anhedonia, there is sleep disturbance, irritability, poor conentration and memory, which all point to a diagnosis of depression. About 1 in 5 women postnatally have a major episode of depression during the first 3 months. Most respond well to psychotherapy, antidepressants, or both. Suicidal ideation should be assessed during consultations, and hospitalisation considered if this is a real risk. Depression in general is common in primary care, affecting some 5-10% of all patients.

64
Q

Mr. Bristow, aged 46, a buying clerk has come at the behest of his wife as he is snoring at night & she cannot bear it any longer. He has given up smoking but it is no better & this has resulted in him putting on 10kgs on his already bulky torso. He is so tired that he sometimes falls asleep at work & he is afraid of being caught by his boss.

All the following patients complain of feeling tired all the time. In addition, they have the following features. Select the most likely diagnosis from the list of options.
A.	RA
B.	Sleep apnoea syndrome
C.	Carcinoma of caecum
D.	AIDS
E.	IDA
F.	Hypothyroidism
G.	Pernicious anaemia
H.	Chronic fatigue syndrome
I.	MS
J.	Depression
A

B. Sleep apnoea syndrome

This is obstructive sleep apnoea and is associated with obesity. Symptoms this patient has such as loud snoring are typical. There may also be gasping during sleep, unrefreshing sleep and excessive sleepiness during the day, which explains why this man is falling asleep at work. Diagnosis is confirmed with polysomnography (also known as a sleep study). Non-invasive interventions include positive airway pressure can be used and there is the option of surgical treatment if indicated. There are many complications of untreated OSA such as MI, stroke and premature death.

65
Q

Mr. James’s tiredness has been coming on over the last 9 months. He is 78 & also complains of pins & needles in the lower limbs in recent weeks & has diarrhoea. His sclera look yellow & his conjunctivae are pale & he has a smooth red tongue. In addition, you detect loss of vibration & joint proprioception. You also think his knee jerks are brisk but you cannot detect any ankle jerks.

All the following patients complain of feeling tired all the time. In addition, they have the following features. Select the most likely diagnosis from the list of options.
A.	RA
B.	Sleep apnoea syndrome
C.	Carcinoma of caecum
D.	AIDS
E.	IDA
F.	Hypothyroidism
G.	Pernicious anaemia
H.	Chronic fatigue syndrome
I.	MS
J.	Depression
A

G. Pernicious anaemia

PA presents with symptoms of anaemia and B12 deficiency. Hence, there is the fatigue and glossitis of anaemia with the neuropathy of B12 deficiency. The Schilling test is now rarely used but does crop up in exams (In this test, IM vitamin B12 is given to saturate stores. Then oral radiolabelled B12 is given and urine is collected over 24 hours. The amount excreted is lower in B12 malabsorption. If this is not corrected by IF the problem is with the ileum and not inadequate IF). Serum B12 levels are useful and APC (anti-parietal cell antibody) can determine whether pernicious anaemia is the cause (but note APC can also be elevated in atrophic gastritis). IF antibody is highly specific for PA but lacks sensitivity compared to APC. Treatment involves supplementation.

66
Q

Mrs. Gray aged 56 has noticed she is feeling cold and tired all the time and has recently become constipated. She also says that she feels depressed. Her husband complains that she is looking ‘frumpy’. She wonders if HRT may help.

All the following patients complain of feeling tired all the time. In addition, they have the following features. Select the most likely diagnosis from the list of options.
A.	RA
B.	Sleep apnoea syndrome
C.	Carcinoma of caecum
D.	AIDS
E.	IDA
F.	Hypothyroidism
G.	Pernicious anaemia
H.	Chronic fatigue syndrome
I.	MS
J.	Depression
A

F. Hypothyroidism

This patient has hypothyroidism. Worldwide, the most common cause is iodine deficiency. Other causes include Hashimoto’s or secondary and tertiary hypothyroidism. It can also result from viral de Quervain’s thyroiditis or postpartum thyroiditis. Symptoms include those mentionedas well as weight gain, menstrual problems in females, dry skin and muscle cramps. Diagnosis is based on measurement of TSH and thyroid hormones. Treatment is by replacement of T4 with or without T3 in combination. If the patient has normal T3 and T4 but mildly elevated TSH, this is described as subclinical hypothyroidism.

67
Q

A 15 year old female with a 2 day history of pyrexia, complains of headaches, feeling weak & a sore throat. On examination, she was pyrexial, had enlarged cervical & axillary lymph nodes & splenomegaly. There was no weight loss or anaemia.

For each question choose the most appropriate answer from the list of options.
A.	Tabes dorsalis
B.	Sleep apnoea
C.	Hypothyroidism
D.	Anaemia
E.	Iatrogenic
F.	Influenza
G.	Diabetes
H.	Chronic fatigue Syndrome
I.	Carcinoma
J.	Infectious mononucleosis
A

J. Infectious mononucleosis

Infectious mononucleosis is caused by EBV and is characterised by fever, pharyngitis and lymphadenopathy. Enlargement of the spleen begins in the first week and lasts 3-4 weeks, occuring in half of all cases. Risk factors for EBV transmission include kissing and sex. A FBC will show an atypical lymphocytosis. Confirmation of IM involves detection of the existence of heterophile antibodies using the Paul Bunnell monospot. A more accurate test is a serological test detecting EBV specific antibodies. Treatment is usually symptomatic but IM carries rare but potentially life threatening complications.

68
Q

A 30 year old male corporate finance banker complains of tiredness of approximately 7 months duration. Tiredness was not substantially alleviated by rest. Tiredness was “severe” & “disabling” & affected his ability at work. Also complained of muscle pain, joint pain & headache. Examination was unremarkable. On direct questioning there was no alcohol or drug misuse, no polydipsia or polyuria. He was not on any medication. His weight had remained at 75kg with no weight loss.

For each question choose the most appropriate answer from the list of options.
A.	Tabes dorsalis
B.	Sleep apnoea
C.	Hypothyroidism
D.	Anaemia
E.	Iatrogenic
F.	Influenza
G.	Diabetes
H.	Chronic fatigue Syndrome
I.	Carcinoma
J.	Infectious mononucleosis
A

H. Chronic fatigue syndrome

This is > 6 months of significant impairing fatigue which is otherwise not explained – the definition of CFS. The aetiology is currently unknown and women are 2-3 times more likely to have CFS. Graduated low-impact exercise and CBT is recommended for all patients. Medication is not effective in CFS. This syndrome is associated with functional impairment which is progressive, depression and OSA.

69
Q

A 55 year old male lorry driver, complains of frequently falling asleep at the wheel of his truck. He has a BMI > 30 & is a heavy smoker. He also admitted to finding it difficult to fall asleep at night & to “putting on a bit of weight” over the past month. Examination was unremarkable.

For each question choose the most appropriate answer from the list of options.
A.	Tabes dorsalis
B.	Sleep apnoea
C.	Hypothyroidism
D.	Anaemia
E.	Iatrogenic
F.	Influenza
G.	Diabetes
H.	Chronic fatigue Syndrome
I.	Carcinoma
J.	Infectious mononucleosis
A

B. Sleep apnoea

This is obstructive sleep apnoea and is associated with obesity which this man has. Smoking is a weak risk factor but strong risk factors, aside from obesity, include male sex, fat neck, fat tongue, chronic snoring and hypothyroidism. Symptoms such as loud snoring are typical. There may also be gasping during sleep, unrefreshing sleep and excessive sleepiness during the day, which explains why this man is falling asleep at the wheel. Diagnosis is confirmed with polysomnography (also known as a sleep study). Non-invasive interventions include positive airway pressure can be used and there is the option of surgical treatment if indicated. There are many untreated complications of OSA such as MI, stroke and premature death.

70
Q

A 45 year old mother of 4 complains of fatigue as well as pain, numbness & tingling in her right hand. She had also noticed her hair had become dry, brittle & thin. She admitted that her weight had increased “quite a bit” over the past few months. On direct questioning she complained of frequently feeling cold & had constipation. On examination she had slow relaxing reflexes & a bradycardia.

vFor each question choose the most appropriate answer from the list of options.
A.	Tabes dorsalis
B.	Sleep apnoea
C.	Hypothyroidism
D.	Anaemia
E.	Iatrogenic
F.	Influenza
G.	Diabetes
H.	Chronic fatigue Syndrome
I.	Carcinoma
J.	Infectious mononucleosis
A

C. Hypothyroidism

Hypothyroidism can cause Carpal Tunnel Syndrome. Other clues in this question include coarse hair, dry/coarse skin, weight gain, cold sensitivity, constipation, slow relaxing reflexes and bradycardia. There may additionally be menstrual irregularities, myalgia, depression and a goitre. Worldwide, the most common cause is iodine deficiency. Diagnosis is based on measurement of TSH and thyroid hormones. Treatment is by replacement of T4 with or without T3 in combination.

71
Q

A 50 year old obese lady with weight loss, tiredness, pruritus vulvae & recurrent boils.

For each patient, choose the most likely cause of the symptom from the list of options.
A.	Hypopituitarism
B.	Diabetes insipidus
C.	Phaeochromocytoma
D.	Acromegaly
E.	Primary hyperparathyroidism
F.	Diabetes mellitus
G.	Cushing's Syndrome
H.	Nelson’s syndrome
I.	Thyrotoxicosis
J.	Myxodema
K.	Addison’s disease
L.	Menopause
M.	Hyperprolactinoma
N.	Porphyria cutanea tarda
A

F. Diabetes mellitus

Pruritus vulvae (an itchy vulva, basically) and recurrent boils suggest this patient is having infections. There is also weight loss and tiredness. This points to T2DM, which can present with yeast, skin and urinary tract infections on top of unintentional weight loss and fatigue.

72
Q

A 20 year old man with marked loss of weight, hyperventilation, drowsiness, polyuria & excessive thirst.

For each patient, choose the most likely cause of the symptom from the list of options.
A.	Hypopituitarism
B.	Diabetes insipidus
C.	Phaeochromocytoma
D.	Acromegaly
E.	Primary hyperparathyroidism
F.	Diabetes mellitus
G.	Cushing's Syndrome
H.	Nelson’s syndrome
I.	Thyrotoxicosis
J.	Myxodema
K.	Addison’s disease
L.	Menopause
M.	Hyperprolactinoma
N.	Porphyria cutanea tarda
A

F. Diabetes mellitus

T1DM presents with polyuria (nocturia is typical), polydipsia (again, at night is typical), weight loss and is indicated by the patient’s young age. The hyperventilation indicates ketoacidosis with respiratory compensation and this patient is currently in DKA. Insulin is needed alongside dietary changes and exercise. Insulin regimes aim to mimic physiological insulin release with a basal-bolus dosing. There is an option between using a pump and having multiple daily injections. It is worth noting that there is a high incidence of diabulimia among young people with T1DM who give themselves less insulin than they need in order to lose weight (they lose weight, ‘look good’ but trash their bodies).

73
Q

A 30 year old lady with weight loss, diarrhoea, tremor, mood swings & a swelling in her neck.

For each patient, choose the most likely cause of the symptom from the list of options.
A.	Hypopituitarism
B.	Diabetes insipidus
C.	Phaeochromocytoma
D.	Acromegaly
E.	Primary hyperparathyroidism
F.	Diabetes mellitus
G.	Cushing's Syndrome
H.	Nelson’s syndrome
I.	Thyrotoxicosis
J.	Myxodema
K.	Addison’s disease
L.	Menopause
M.	Hyperprolactinoma
N.	Porphyria cutanea tarda
A

I. Thyrotoxicosis

This woman has hyperthyroidism (thyrotoxicosis). Symptoms include those mentioned and heat intolerance, sweating and tachycardia. In countries where sufficient iodine intake is not an issue, Graves’ disease is the most common cause of hyperthyroidism which presents with a diffuse goitre. Treatment aims to normalise thyroid function and is achieved by radioactive iodine, antithyroid medications or with surgery. Symptomatic therapy is given with beta blockers such as propranolol.

74
Q

A 50 year old lady with weight gain, constipation, depression, heavy periods & a hoarse voice.

For each patient, choose the most likely cause of the symptom from the list of options.
A.	Hypopituitarism
B.	Diabetes insipidus
C.	Phaeochromocytoma
D.	Acromegaly
E.	Primary hyperparathyroidism
F.	Diabetes mellitus
G.	Cushing's Syndrome
H.	Nelson’s syndrome
I.	Thyrotoxicosis
J.	Myxodema
K.	Addison’s disease
L.	Menopause
M.	Hyperprolactinoma
N.	Porphyria cutanea tarda
A

J. Myxodema

This is hypothyroidism (myxoedema). Coarse hair, dry/coarse skin, weight gain, cold sensitivity, constipation, slow relaxing flexes, bradycardia, menstrual irregularities, myalgia, depression and a goitre may all be found. Diagnosis is based on measurement of TSH and thyroid hormones. Treatment is by replacement of T4 with or without T3 in combination.

75
Q

A 40 year old man with limb pains, epigastric pain with vomiting, colicky pain in his left loin. He has been feeling low & depressed recently & also complained of excessive thirst, nocturia & loss of appetite.

For each patient, choose the most likely cause of the symptom from the list of options.
A.	Hypopituitarism
B.	Diabetes insipidus
C.	Phaeochromocytoma
D.	Acromegaly
E.	Primary hyperparathyroidism
F.	Diabetes mellitus
G.	Cushing's Syndrome
H.	Nelson’s syndrome
I.	Thyrotoxicosis
J.	Myxodema
K.	Addison’s disease
L.	Menopause
M.	Hyperprolactinoma
N.	Porphyria cutanea tarda
A

E. Primary hyperparathyroidism

There is autonomous PTH production in primary HPT which causes deranged calcium metabolism. Biochemistry will show elevated serum calcium and inappropriate elevation of PTH. Depression, cognitive changes, change in sleep (possibly due to change in circadian rhythm) and myalgia are all common complaints. This patient’s bone pain is a common complaint which may occur with osteoporosis. Osteoporosis occurs due to excess PTH causing bone resorption (osteoclasts are stimulated). This patient also has a kidney stone in the left loin due to hypercalciuria. Other symptoms of hypercalcaemia are present including nocturia and the patient also seems to have pancreatitis. I agree, a patient in real life is unlikely to be unlucky enough to present with such a host of symptoms. The only definitive cure is a parathyroidectomy although complications of this procedure include hypocalcaemia, injury to the recurrent laryngeal nerve, bleeding and a pneumothorax.

76
Q

A 50 year old asthmatic lady with painful muscles, bruising, purple marks on her skin. She sustained a laceration on her left which took 2 weeks to heal. On examination she looked fat, especially around her face & trunk.

For each patient, choose the most likely cause of the symptom from the list of options.
A.	Hypopituitarism
B.	Diabetes insipidus
C.	Phaeochromocytoma
D.	Acromegaly
E.	Primary hyperparathyroidism
F.	Diabetes mellitus
G.	Cushing's Syndrome
H.	Nelson’s syndrome
I.	Thyrotoxicosis
J.	Myxodema
K.	Addison’s disease
L.	Menopause
M.	Hyperprolactinoma
N.	Porphyria cutanea tarda
A

G. Cushing’s syndrome

This asthmatic is likely on steroids (inhaled, or oral). This can lead to iatrogenic Cushing’s syndrome. A low dose dexamethasone suppresion test can be done, or a 24 hour urinary free cortisol collection to diagnose Cushing’s syndrome.

77
Q

A 35 year old lady with weight loss, poor appetite, abdominal pains & irregular periods. On examination she had white patches on her face & dark colouring in the palms of her hands.

For each patient, choose the most likely cause of the symptom from the list of options.
A.	Hypopituitarism
B.	Diabetes insipidus
C.	Phaeochromocytoma
D.	Acromegaly
E.	Primary hyperparathyroidism
F.	Diabetes mellitus
G.	Cushing's Syndrome
H.	Nelson’s syndrome
I.	Thyrotoxicosis
J.	Myxodema
K.	Addison’s disease
L.	Menopause
M.	Hyperprolactinoma
N.	Porphyria cutanea tarda
A

K. Addison’s disease

Hyperpigmentation in the palmar creases points towards Addison’s disease. Hyperpigmentation due to excess ACTH production can be mucosal or cutaneous and is more pronounced in the palms, knuckles and around scars. MSH is a byproduct of the production of ACTH from the cleavage of POMC. Sodium is low and potassium elevated. Patients like this one will also present with weight loss. There may also be abdominal pain as a GI complaint. Vitiligo is an autoimmune condition and patients with autoimmune diseases are at increased risk of developing Addison’s. Diagnosis of Addison’s can be made on an ACTH stimulation test (synacthen test) whereby serum cortisol remains low despite the administration of synthetic ACTH. In an emergency, treatment should not be delayed by diagnostic testing.

78
Q

A 40 year old hypertensive man presented with weight loss. He also admits to episodes of headaches, palpitations & sweating. Examination during one of these episodes showed sugar in the urine & a blood pressure of 240/120.

For each patient, choose the most likely cause of the symptom from the list of options.
A.	Hypopituitarism
B.	Diabetes insipidus
C.	Phaeochromocytoma
D.	Acromegaly
E.	Primary hyperparathyroidism
F.	Diabetes mellitus
G.	Cushing's Syndrome
H.	Nelson’s syndrome
I.	Thyrotoxicosis
J.	Myxodema
K.	Addison’s disease
L.	Menopause
M.	Hyperprolactinoma
N.	Porphyria cutanea tarda
A

C. Phaeochromocytoma

Phaeochromocytomas presents with paroxysmal episodes of palpitations, anxiety, excessive sweating, pallor and hypertension. The patient may complain of headaches. It can be inherited in MEN2, von Hippel-Lindau syndrome and NF1. Diagnosis is based on raised urinary and serum catecholamines, metanephrines and normetanephrines. 24 hour urinary VMA will be elevated. CT is used to localise the tumour. Treatment includes medical with the use of phenoxybenzamine, phentolamine and surgical options. Surgical excision is carried out under alpha and beta blockade to protect against the release of catecholamines into circulation when the tumour is being manipulated. The 10% rule is often quoted: 10% are bilateral, 10% malignant, 10% extraadrenal and 10% hereditary.

79
Q

A 71 year old lady presented with depression & personality disorder which was first noticed during long term psychiatric care. On examination there was marked alopecia & loss of facial contours.

Select the single most appropriate treatment for the following patients.
A.	Hydrocortisone
B.	Gliclazide
C.	Combined FSH/LH
D.	Octreotide
E.	Thyroxine
F.	Carbimazole
G.	Surgery
H.	Ethyloestradiol
I.	Alfacalcidol
J.	GnRH subcutaneous infusion pumps
K.	Propanolol
L.	Oral sugar
M.	Spironolactone
A

E. Thyroxine

This is hypothyroidism and this lady will need replacement T4 (levothyroxine), with or without T3. Treatment is lifelong and the main complication is over-replacement which can cause AF and osteoporosis. The dose must be increased in pregnancy, where requirements of T3 and T4 increase. A lower dose of levothyroxine is recommended if the patient has coronary artery disease as it can exacerbate angina.

80
Q

A 45 year old lady came to the GP with worries about recent facial hair growth & acne. She’s noticed a steady increase in her weight recently. A blood test was taken which showed very high ACTH levels.

Select the single most appropriate treatment for the following patients.
A.	Hydrocortisone
B.	Gliclazide
C.	Combined FSH/LH
D.	Octreotide
E.	Thyroxine
F.	Carbimazole
G.	Surgery
H.	Ethyloestradiol
I.	Alfacalcidol
J.	GnRH subcutaneous infusion pumps
K.	Propanolol
L.	Oral sugar
M.	Spironolactone
A

G. Surgery

This is a pituitary adenoma secreting ACTH (i.e. Cushing’s disease) which will have to be definitively excised by transsphenoidal pituitary adenomectomy as the first line intervention. Adrenal aetiology in Cushing’s syndrome would cause a suppressed ACTH. Think about the HPA axis and draw it out if you find it difficult to get your head around this.

81
Q

A 21 year old male presented with fatigue, weight loss, constipation & postural hypotension. On examination he had an abnormally coloured scar from his recent appendectomy.

Select the single most appropriate treatment for the following patients.
A.	Hydrocortisone
B.	Gliclazide
C.	Combined FSH/LH
D.	Octreotide
E.	Thyroxine
F.	Carbimazole
G.	Surgery
H.	Ethyloestradiol
I.	Alfacalcidol
J.	GnRH subcutaneous infusion pumps
K.	Propanolol
L.	Oral sugar
M.	Spironolactone
A

A. Hydrocortisone

This patient has Addison’s disease. Hyperpigmentation due to excess ACTH production can be mucosal or cutaneous and is more pronounced in the palms, knuckles and around scars. There is also fatigue, weight loss and postural hypotension. Treatment is with oral GC and MC replacement for life. Hydrocortisone is a glucocorticoid which can be given, and fludrocortisone is a mineralocorticoid which can be given.

82
Q

A lady came to her GP due to problems trying to conceive. She says her periods have been unpredictable for the past 2 years. She’s been suffering from double vision recently. Other than that she’s been fine. On examination her eyes appeared to bulge out & had swelling periorbitally.

Select the single most appropriate treatment for the following patients.
A.	Hydrocortisone
B.	Gliclazide
C.	Combined FSH/LH
D.	Octreotide
E.	Thyroxine
F.	Carbimazole
G.	Surgery
H.	Ethyloestradiol
I.	Alfacalcidol
J.	GnRH subcutaneous infusion pumps
K.	Propanolol
L.	Oral sugar
M.	Spironolactone
A

F. Carbimazole

This is Graves’ disease and this patient is describing Graves’ ophthalmopathy with diplopia, exopthalmos and periorbital puffiness. The menstrual problems this woman reports can also be seen in hyperthyroidism. Graves’ can be treated with antithyroid drugs which include carbimazole. Carbimazole (similar drugs include PTU and methimazole, which is what carbimazole is converted to) gradually decreases T3 and T4 output by inhibiting thyroperoxidase which iodinates tyrosyl residues in thyroglobulin to give T3 and T4 precursors. A rare adverse effect of carbimazole is agranulocytosis. Rashes and joint pain can also occur. The response can take several weeks due to thyroid stores and the long half life of T4.

83
Q

A 45 year old obese man complains of being very tired. He puts it down to lack of sleep due to having to go to the toilet a lot during the night. He finds he has blurry eyes often in the morning.

Select the single most appropriate treatment for the following patients.
A.	Hydrocortisone
B.	Gliclazide
C.	Combined FSH/LH
D.	Octreotide
E.	Thyroxine
F.	Carbimazole
G.	Surgery
H.	Ethyloestradiol
I.	Alfacalcidol
J.	GnRH subcutaneous infusion pumps
K.	Propanolol
L.	Oral sugar
M.	Spironolactone
A

B. Gliclazide

This patient has DM and the only oral hypoglycaemic on the list is gliclazide. It is a sulphonylurea, and other drugs in this class include tolbutamide, glibenclamide and glipizide. It acts to increase insulin release from beta cells by acting on the sulphonylurea receptor causing potassium channels to close. This causes depolarisation and calcium entry via VGCCs which stimulates insulin secretion by exocytosis. Side effects include hypoglycaemia and weight gain.

84
Q

A 58 year old woman with a history of frequent headaches, general tiredness, polydipsia & an increase in shoe size. She complained of a slight loss of peripheral vision in both her eyes. BMI is 31.

Match the patient description with one of the diagnoses.
A.	Acromegaly
B.	Myxoedema
C.	Addison’s disease
D.	De Quervain's thyroiditis
E.	Cretinism
F.	Graves disease
G.	Dwarfism
H.	Cushing’s syndrome
I.	Plummer's disease
J.	Diabetes insipidus
K.	Gigantism
L.	Diabetes mellitus
A

A. Acromegaly

Acromegaly is caused by excess growth hormone and is most often due to a pituitary adenoma. The diagnosis is often made late as the symptoms are insidious in onset. This patient has an increase in shoe size. There may also be an enlarged nose, prognathism, soft tissue changes and organomegaly, all as a result of excess GH/IGF-1. The visual impairment is due to the pituitary adenoma putting pressure on the optic chiasm. Tumour mass effect may also cause headaches, there may also be hypopituitarism from stalk compression and CN palsies. IGT and DM are associated and this may account for the polydipsia (and possible polyuria). There may also be altered sexual function, Carpal Tunnel Syndrome and joint dysfunction. Treatment is either sugical with transsphenoidal surgery, or medical (if the tumour cannot be resected/completely resected) with a somatostatin analogue like octreotide and an adjunctive dopamine agonist like cabergoline. If the patient does not respond to SSAs then pegvisomant which is a GH receptor antagonist can be used, although it is very costly. Gigantism occurs as a result of excess GH during childhood.

85
Q

A 25 year old lady presenting with difficulty sleeping, irritability, history of palpitations, weight loss & sweating over the past year. 2 aunts had neck operations. She had a swelling in her neck, pretibial myxoedema & exopthalmos. Pulse 112 beats per min, BP 106/70.

Match the patient description with one of the diagnoses.
A.	Acromegaly
B.	Myxoedema
C.	Addison’s disease
D.	De Quervain's thyroiditis
E.	Cretinism
F.	Graves disease
G.	Dwarfism
H.	Cushing’s syndrome
I.	Plummer's disease
J.	Diabetes insipidus
K.	Gigantism
L.	Diabetes mellitus
A

F. Graves’ disease

The goitre combined with peripheral signs points to Graves’ disease as Graves’ is the only cause of hyperthyroidism which presents peripheral signs like pretibial myxoedema and exopthalmos. The irritability, palpitations, weight loss and sweating all indicate hyperthyroidism. The cause is indicated from the peripheral stigmata. The possible FH of autoimmune thyroid disease is a strong risk factor.

86
Q

A 32 year old woman with progressive tiredness over the last 2 years since the birth of her daughter. Had been fired from her job for being too slow. Admits to being constipated & having heavy periods. No family history. Pulse 54, BP 110/75.

Match the patient description with one of the diagnoses.
A.	Acromegaly
B.	Myxoedema
C.	Addison’s disease
D.	De Quervain's thyroiditis
E.	Cretinism
F.	Graves disease
G.	Dwarfism
H.	Cushing’s syndrome
I.	Plummer's disease
J.	Diabetes insipidus
K.	Gigantism
L.	Diabetes mellitus
A

B. Myxoedema

This patient’s symptoms are consistent with hypothyroidism. Worldwide, the most common cause is iodine deficiency. Other causes include Hashimoto’s or secondary and tertiary hypothyroidism. It can also result from viral de Quervain’s thyroiditis or postpartum thyroiditis. Diagnosis is based on measurement of TSH and thyroid hormones. Treatment is by replacement of T4 with or without T3 in combination. If the patient has normal T3 and T4 but mildly elevated TSH, this is described as subclinical hypothyroidism.

87
Q

A 55 year old female complains of worsening obesity over the past few years, she also has a history of high blood pressure, diabetes, osteoporosis & proximal myopathy. She has a round face & multiple bruises & thin skin.

Match the patient description with one of the diagnoses.
A.	Acromegaly
B.	Myxoedema
C.	Addison’s disease
D.	De Quervain's thyroiditis
E.	Cretinism
F.	Graves disease
G.	Dwarfism
H.	Cushing’s syndrome
I.	Plummer's disease
J.	Diabetes insipidus
K.	Gigantism
L.	Diabetes mellitus
A

H. Cushing’s syndrome

This patient has presented with the classic signs and symptoms of Cushing’s syndrome. There are signs of hypercortisolism: obesity (central), hypertension, DM (due to glucose intolerance), osteoporosis, proximal myopathy, easy bruising due to thin skin and SC tissue and a moon face. Other signs and symptoms include acne, low libido, buffalo hump (dorsocervical fat pads), striae, facial plethora and menstrual irregularities. A diagnostic test is needed such as 48 hour 2mg dexamethasone suppression testing. ACTH should be measured and if suppressed then further testing should focus on the adrenals. If not suppressed then pituitary or ectopic disease should be suspected. In most cases, surgical resection is the treatment of choice, whether a pituitary or adrenal adenoma is to blame.

88
Q

Soon after birth a baby boy is transferred to ITU with symptoms of dehydration, although he is producing copious urine. Once he has been liberally supplied with fluids he is no longer dehydrated. However when he is given a test dose of DDAVP there is no effect and still polyuria. Ketones or glucose are not found in the urine.

Match the patient description with one of the diagnoses.
A.	Acromegaly
B.	Myxoedema
C.	Addison’s disease
D.	De Quervain's thyroiditis
E.	Cretinism
F.	Graves disease
G.	Dwarfism
H.	Cushing’s syndrome
I.	Plummer's disease
J.	Diabetes insipidus
K.	Gigantism
L.	Diabetes mellitus
A

J. Diabetes insipidus

This is nephrogenic diabetes insipidus where there is polydipsia, polyuria, and hypotonic urine in large quantities. Central DI is due to defective synthesis or release of AVP and will hence respond to DDAVP (desmopressin). Nephrogenic DI, on the other hand, occurs due to renal insensitivity to AVP. Hence, it will not respond to DDAVP and trearment is with adequate fluids, salt restriction and thaizide diuretics (paradoxically). This boy could have an inherited form of nephrogenic DI with a mutation in the AVP receptor pathway in the collecting duct. There may also be AQP2 receptor mutations. Other risks for nephrogenic DI include lithium use, CKD and chronic hypercalcaemia or hypokalaemia.

89
Q

A woman has been trying for a baby for a number of years now to no avail, she has suffered two miscarriages in the past. Her only past medical history of note is a ‘dodgy’ rhythm she has received treatment for for years.

What is the most likely diagnosis?
A. Addisonian crisis
B. External neck irradiation
C. Grave's disease
D. Myxoedema coma
E. Amiodarone
F. Hashimoto's thyroiditis
G. De Quervains thyroiditis
H. Thyroxine abuse
A

E. Amiodarone

Any patient with a history of miscarriage should be investigated for thyroid dysfunction especially hypothyroidism. In this lady’s case she has been taking amiodarone as an anti-arrhythmic long term, Amiodarone is one of several drugs that can cause hyper/hypothyroidism.

90
Q

A 75 yo man with a 50 pack year history complains of weight gain and tingling in his hands especially at night, he also feels generally lethargic. The last time he was in hospital he was receiving treatment after having something ‘nasty’ cut out of his tongue.

What is the most likely diagnosis?
A. Addisonian crisis
B. External neck irradiation
C. Grave's disease
D. Myxoedema coma
E. Amiodarone
F. Hashimoto's thyroiditis
G. De Quervains thyroiditis
H. Thyroxine abuse
A

B. External neck irradiation

The man’s history points to oral cancer, for which he would have received a wide local excision most likely. This would have been followed up with radiotherapy to the local nodes and tissues of the neck. In this gentleman the radiation has caused his thyroid to become underactive hence the lethargy and tiredness. The tingling in his hands at night is a classic presentation of Carpal tunnel syndrome (median nerve compression at the flexor retinaculum) which has an increased incidence in patients with hypothyroidism.

91
Q

A middle aged women is seen in clinic complaining that she is depressed and that her taste is impaired, she also has muscle aches. The dr reviews her bloodwork and sees her cholesterol and creatine kinase are elevated. He examines her neck and feels a firm, rubbery structure overlying the trachea that moves on swallowing.

What is the most likely diagnosis?
A. Addisonian crisis
B. External neck irradiation
C. Grave's disease
D. Myxoedema coma
E. Amiodarone
F. Hashimoto's thyroiditis
G. De Quervains thyroiditis
H. Thyroxine abuse
A

F. Hashimoto’s thyroiditis

This patient has signs of hypothyroidism: depressed mood and impaired taste. Also she has hypothyroid myopathy, evidenced both symptomatically and biochemically by the elevated creatine kinase. Further to this it is worth noting that hypothyroidism has other effects that can be seen on bloodwork. Amongst these are anaemia and hypercholesterolaemia. It is the thyroid that moves on swallowing and on examination hers is rubbery and firm, this is typical of hashimoto’s thyroiditis which is one of the leading causes of hypothyroidism in the UK. De Quervain’s is not an option owing to lack of preceding viral infection and no tenderness over the neck.

Hashimoto’s thyroiditis is an autoimmune thyroiditis with associations with HLA -DR5 (and some conditions such as Turner’s syndrome, T1DM and pernicious anemia) that shows T - lymphocyte infiltration of the gland and reactive antibodies against the TSH receptor and less commonly Thyroglobulin. It is most prevalent in middle aged women and is a prominent cause of goitre, due to the infiltration of the gland and elevated TSH levels (since T4 production is decreased). Note that at presentation most patients are euthyroid, although around half end up hypo due to destruction of the thyroid. It is possible to be toxic with hasimotos. Hashitoxicosis however is rare and can be transient. Treatment is generally thyroxine T4 given at doses sufficient to suppress endogenous T4 production and minimise TSH, this is done to lessen the risk of a goitre forming and subsequently growing.

92
Q

An 80 year old woman is brought into hospital following a house visit. She was initially confused, but in hospital she is found to be hypoventilating, her bloods show low BMs and hyponatraemia. Drs order hydrocortisone and liothyronine sodium.

What is the most likely diagnosis?
A. Addisonian crisis
B. External neck irradiation
C. Grave's disease
D. Myxoedema coma
E. Amiodarone
F. Hashimoto's thyroiditis
G. De Quervains thyroiditis
H. Thyroxine abuse
A

D. Myxoedema coma

This patient is at the severe end of the hypothyroid spectrum. She has depressed level of consciousness (this can descend into frank coma) and is hypoventilating, other features that can present in this serious complication are bradycardia, and hyponatraemia. Further to this a massive proportion are hypothermic (core temp < 35 degrees celsius) although there is absence of shivering. The mortality is high..around 50%. This is a diagnosis not to be missed so anyone with depressed consciousness should have thyroid function tests performed, especially the elderly. NB: Always look for the cause, often there is a precipitant..be it a cold home, narcotics or infection. Mx : she would need supportive measure to correct her respiratory status and iv fluids, body temperature must also be maintained. Replacement of thyroid hormone is by administration of bolus T3 ideally (the drug is liothyronine). This is followed up by oral T3, if this is contra-indiciated thyroxine can also be used. Since she has also received glucocorticoid replacement for low BMs and hyponatraemia it may be reasonable to assume her hypothyroidism is pituitary in origin.

93
Q

A teenage girl is seen in clinic for palpitations, she looks extremely thin but does not bring this up with the doctor. Her pulse is irregularly irregular her mother is concerned she may have inherited her gland problems.

What is the most likely diagnosis?
A. Addisonian crisis
B. External neck irradiation
C. Grave's disease
D. Myxoedema coma
E. Amiodarone
F. Hashimoto's thyroiditis
G. De Quervains thyroiditis
H. Thyroxine abuse
A

H. Thyroxine abuse

This patient is not worried by her weight loss as it is self induced, she has no features that confirm grave’s disease though she is thyrotoxic. The mother’s condition is not stated but you can deduce that the girl has access to thyroxine which she is abusing for it’s metabolic properties.

94
Q

A 48 yo woman is seen in clinic complaining of a non-painful lump in her neck that has been getting larger. The Dr notes that her voice sounds hoarse and she has been having difficulty swallowing her food. O/E there is a large ‘iron’ hard lump on the left side of her neck that seems to be fixed to the underlying structures. The dr thinks Tamoxifen might help.

What is the most likely diagnosis?
A. Grave's disease 
B. Riedel's thyroiditis 
C. Subacute lymphocytic thyroiditis 
D. MEN 2B 
E. Late De Quervain's thyroiditis 
F. Medullary thyroid cancer 
G. Early De Quervain's thyroiditis
A

B. Riedel’s thyroiditis

Riedel’s is typically described as Iron hard or ‘woody’. It is of an autoimmune origin and 25-50% of those affected are hypothyroid. The presenation is of a painless lump in the neck that can also cause local compressive symptoms. ie. Tracheal/oesophageal compression..stridor etc. Since the lump is hard and craggy it can be easily mistaken for carcinoma, however there is no lymphadenopathy in this patient or general cancer symptoms. Riedel’s can be associated with sclerosing cholangitis and peculiarly responds to Tamoxifen, there can also be use for steroids.
Patient 2: anxious and with opthalmoplegia, immediately you are thinking thyrotoxic and more specifically Grave’s as opthalmopathy is almost always associated with this particular pathology. The anti TSH receptor antibodies are a finding in Graves and they are likely responsible for the opthalmopathy and pre-tibial myxoedema sometimes seen. Thyroid stimulating immunoglobulin (TSI) is also implicated. A Radio-idoine uptake scan of the thyroid would show a ‘hot’ gland with diffuse symmetrical enlargement.

95
Q

An anxious looking lady presents to the endocrinology clinic to receive her test results. She is positive for anti-TSH receptor antibodies, eye movement testing reveals some opthalmoplegia.

What is the most likely diagnosis?
A. Grave's disease 
B. Riedel's thyroiditis 
C. Subacute lymphocytic thyroiditis 
D. MEN 2B 
E. Late De Quervain's thyroiditis 
F. Medullary thyroid cancer 
G. Early De Quervain's thyroiditis
A

A. Grave’s disease

Anxious and with opthalmoplegia, immediately you are thinking thyrotoxic and more specifically Grave’s, as opthalmopathy is almost always associated with this particular pathology. The anti TSH receptor antibodies are a finding in Graves and they are likely responsible for the opthalmopathy and pre-tibial myxoedema sometimes seen. Thyroid stimulating immunoglobulin (TSI) is also implicated. A Radio-iodine uptake scan of the thyroid would show a ‘hot’ gland with diffuse symmetrical enlargement.

96
Q

A man has recently recovered from a sore throat and now has a painful lump in his neck. FNA was performed and showed giant cells on histology. A radioiodine uptake scan was ‘cold’. The patient feels anxious and has a racing heart: he was prescribed Propranolol.

What is the most likely diagnosis?
A. Grave's disease 
B. Riedel's thyroiditis 
C. Subacute lymphocytic thyroiditis 
D. MEN 2B 
E. Late De Quervain's thyroiditis 
F. Medullary thyroid cancer 
G. Early De Quervain's thyroiditis
A

G. Early De Quervain’s thyroiditis

Preceding viral infection already points towards De Quervain’s. The fine needle aspirate shows giant cells which are collections of macrophages which form in response to infection or foreign bodies. In this case the gentleman is anxious suggesting he is thyrotoxic, this is reinforced by the low uptake on radio iodine scanning. In the early stages of De Quervain’s thyroiditis viral infiltration of the gland leads to destruction and release of preformed T4/T3, this leads to thyrototoxicosis. There is low uptake as the gland is responding to feedback, the raised serum T4 is a consequence of gland destruction not increased synthesis. As a result of this there is a second phase where the endogenous T4/T3 stores are depleted and the patient becomes hypothyroid. Early management is symptomatic and generally supportive e.g. beta blockade.

97
Q

An unusually tall middle aged lady seen in clinic has a large painless lump in her neck that is fixed. The Dr also notes cervical lymphadenopathy and yellow white nodules on her lips. Pembertons sign is positive and a thoracic inlet scan confirms the compression. Her medical history includes surgery on her aortic valve and some work on her lens.

What is the most likely diagnosis?
A. Grave's disease 
B. Riedel's thyroiditis 
C. Subacute lymphocytic thyroiditis 
D. MEN 2B 
E. Late De Quervain's thyroiditis 
F. Medullary thyroid cancer 
G. Early De Quervain's thyroiditis
A

D. MEN 2B

This fixed lump is accompanied by lymphadenopathy, warning bells go off that this may be malignant. There are local invasion signs also as confirmed by pemberton’s sign.

Pemberton’s sign - I put this into a thyroid exam, It can equally be justified in a respiratory examination. Basically you ask the patient to lift their arms straight above their head as high as possible and then listen for inspiratory stridor, look for facial flushing or distension of veins acorss the neck. You can ask the patient to tell you if they fell light headed when performing this test.

It basically looks to see if there is evidence of SVC (superior venae cava) syndrome. If there is a large goitre or apical lung tumour for example impinging on the SVC, lifting the arms will lead to compression and cause congestion of blood in the head. This can be seen as a plethoric face, a cyanosed face, the patient may experience nausea and headache etc.

Positive Pemberton’s sign signifies thoracic inlet obstruction causes of which include:

  • Retrosternal goitre
  • Lung carcinomas
  • Aortic Aneurysms
    Thoracic inlet scan shows that a goitre is compressing structures such as the Superior vena cava. This seems to be maligant..Medullary thyroid carcinoma. however the rest of the patient must be taken into account..she has a marfanoid appearance and suggestion of other features of the syndrome. the lens work would be to correct dislocation, and the aortic valve work no doubt was to correct aortic root dilatation and valve incompetency seen in Marfan’s. The nodules are neurofibromatoses typical of MEN 2b.
98
Q

A man is seen in the endocrinology clinic for his thyroid condition. He has the ‘thyroid stare’ and closer inspection of his eyes shows the optic disc is pale and swollen.

What is the most appropriate treatment?
A. Propranolol
B. Stop treatment
C. Surgical decompression
D. Radioiodine
E. Potassium Iodide
F. Propylthiouracil
G. Carbimazole
A

C. Surgical decompression

There is significant retroorbital oedema and fibrosis around the optic nerve, decompressive surgery should help relieve this and prevent further atrophy.

99
Q

A young woman has been diagnosed with Grave’s disease and has been receiving oral treatment for the past few weeks. She has now developed a sore throat and a nasty cold.

What is the most appropriate treatment?
A. Propranolol
B. Stop treatment
C. Surgical decompression
D. Radioiodine
E. Potassium Iodide
F. Propylthiouracil
G. Carbimazole
A

B. Stop treatment

The sore throat should be treated with a high index of suspicion as it might warn of neutropenic agranulocytosis, a serious side effect of Carbimazole. Bloods should be taken to determine if this has occurred and if so the drug should be stopped.

100
Q

A man has dysphagia which has been getting progressively worse, he has also lost weight. He is submitted for thyroidectomy. A few weeks later at a checkup it is noticed that his thyroglobulin levels are creeping up again.

What is the most appropriate treatment?
A. Propranolol
B. Stop treatment
C. Surgical decompression
D. Radioiodine
E. Potassium Iodide
F. Propylthiouracil
G. Carbimazole
A

D. Radioiodine

A malignant thyroid has been removed (weight loss, dysphagia) and the thyroglobulin is mentioned as it is useful as a marker of neoplastic activity in patients with these malignancies (much like PSA in prostate Ca), carcinoembryonic antigen : CEA may also be used in the same way. Recurrence or suggestion of should be treated promptly with followup irradiation.

101
Q

A 21 yo man has had a sore throat recently and now developed a very tender gland on his neck which is quite erythematous. He has also been feeling quite jittery and anxious, he complains of thumping in his chest.

What is the most appropriate treatment?
A. Propranolol
B. Stop treatment
C. Surgical decompression
D. Radioiodine
E. Potassium Iodide
F. Propylthiouracil
G. Carbimazole
A

A. Propranolol

De Quervain’s is caused by infection. It is early stage hence he is thyrotoxic. Management is mainly supportive: analgesia and beta blockade for the palpitations and anxiety.

102
Q

A woman has a firm diffuse goitre, she’s feeling tired.

What is the most likely diagnosis?
A. Solitary toxic adenoma
B. Follicular carcinoma
C. Medullary cell carcinoma
D. Papillary carcinoma
E. Multinodular goitre
F. Grave's disease
G. Hashimoto's thyroiditis
H. De Quervain's thyroiditis
A

G. Hashimoto’s thyroiditis

103
Q

A woman has a lumpy swelling over the front of her neck and has been complaining of difficulty swallowing. She is euthyroid.

What is the most likely diagnosis?
A. Solitary toxic adenoma
B. Follicular carcinoma
C. Medullary cell carcinoma
D. Papillary carcinoma
E. Multinodular goitre
F. Grave's disease
G. Hashimoto's thyroiditis
H. De Quervain's thyroiditis
A

E. Multinodular goitre

104
Q

A man has a history of weight loss and his voice has recently hoarsened. He has been complaining of pain in his leg also. The Dr reviews his blood work and highlights a raised calcitonin.

What is the most likely diagnosis?
A. Solitary toxic adenoma
B. Follicular carcinoma
C. Medullary cell carcinoma
D. Papillary carcinoma
E. Multinodular goitre
F. Grave's disease
G. Hashimoto's thyroiditis
H. De Quervain's thyroiditis
A

C. Medullary cell carcinoma

105
Q

A woman presents in clinic with dizzy spells and abdominal pain, her BP is taken sitting 130/80 and then standing 105/75. On examination of her mouth it’s noted that she has darkening of her gums.

What is the most likely diagnosis?
A. Adrenal adenoma
B. Addisonian crisis
C. Sheehan's syndrome
D. Tuberculosis
E. Addison's disease
F. Pituitary apoplexy
G. Cushing's disease
H. Nelson's syndrome
I. Conn's Adenoma
J. Ectopic ACTH producing tumour
K. Drug withdrawal
L. Diabetes Insipidus
A

E. Addison’s disease

This patient has symptoms of adrenal insufficiency. Her Systolic BP falls by more than 20mmHg on standing which means she has postural hypotension. Hyperpigmentation of the gingiva and buccal mucosa suggests high ACTH output which in this patient would be due to primary adrenal insufficiency: Addison’s disease. The condition is probably of autoimmune origin in this girl and a positive finding may be 21 hydroxylase auto-antibodies.

106
Q

A man comes to clinic complaining of weight gain and a cut on his leg that just won’t heal. He has various blood tests performed including a CRH (corticotrophin releasing hormone) stimulation test. His ACTH and cortisol are both shown to rise.

What is the most likely diagnosis?
A. Adrenal adenoma
B. Addisonian crisis
C. Sheehan's syndrome
D. Tuberculosis
E. Addison's disease
F. Pituitary apoplexy
G. Cushing's disease
H. Nelson's syndrome
I. Conn's Adenoma
J. Ectopic ACTH producing tumour
K. Drug withdrawal
L. Diabetes Insipidus
A

G. Cushing’s disease

This man has weight gain and impaired wound healing, already you should be thinking Cushing’s or diabetes. The blood test used here is being performed to differentiate between Cushing’s disease and Cushing’s syndrome.

10ug ovine or human CRH is administered to a patient who has fasted at least 4 hrs. pre test and serial post test ACTH and cortisol measurements are taken.

In pituitary driven Cushing’s disease the cortisol can be manipulated, that is to say exogenous CRH will cause a recordable increase in serum cortisol by the 2hr mark (and ACTH). However if the source of the excess cortisol is an ectopic ACTH producing tumour for example, this is not subject to any feedback mechanisms and so no change in serum cortisol will be seen on giving CRH.

107
Q

A dishevelled man local to the area is found collapsed on the street. In A+E his blood work reveals low sodium and a raised potassium, he is oliguric, tachcyardic and his BMs are running dangerously low. Chest X ray shows pathological upper lobe changes and abdominal films show areas of calcification.

What is the most likely diagnosis?
A. Adrenal adenoma
B. Addisonian crisis
C. Sheehan's syndrome
D. Tuberculosis
E. Addison's disease
F. Pituitary apoplexy
G. Cushing's disease
H. Nelson's syndrome
I. Conn's Adenoma
J. Ectopic ACTH producing tumour
K. Drug withdrawal
L. Diabetes Insipidus
A

B. Addisonian crisis

Though this patient does have TB, Addisonian crisis is the acute condition that has caused him to collapse.

He is shown to be hyponataraemic and hyperkalaemic. This picture fits adrenal failure well, the raised potassium is a result of mineralocorticoid deficiency leading to decreased K+ excretion.

The CXR is hinting at upper lobe fibrosis as seen in patients who have had Tuberculosis, and further the abdo film shows calcification of the adrenals bilaterally which is the radiographic appearance of TB infiltration of the adrenal galnds.

This man has Addison’s caused by TB. Furthermore he is found collapsed with low blood glucose. He has signs of shock…Tachycardia and oliguria so he is in fact in Addisonian crisis which should be treated promptly with gluocorticoid replacement and dextrose, this man has evidence of mineralocorticoid deficiency also so fludrocortisone should be coadministered.

108
Q

Woman who has been complaining of worsening loss of peripheral vision for months suddenly takes a turn for the worse. She has an intense headache come on suddenly at home, by the time she is driven to hospital she has vomited a number of times and now complains of double vision. Her blood pressure also falls drastically. She is managed initially with hydrocortisone I.V. and saline. Thyroxine follows. A lumbar puncture is requested, it shows Xanthochromia.

What is the most likely diagnosis?
A. Adrenal adenoma
B. Addisonian crisis
C. Sheehan's syndrome
D. Tuberculosis
E. Addison's disease
F. Pituitary apoplexy
G. Cushing's disease
H. Nelson's syndrome
I. Conn's Adenoma
J. Ectopic ACTH producing tumour
K. Drug withdrawal
L. Diabetes Insipidus
A

F. Pituitary apoplexy

This patient has bitemporal hemianopia which has been worsening, and this should spark thoughts of a growing pituitary adenoma. But then she presents with an acute change. With a tumour a sudden change is usually vascular.. in this case the patient is suffering pituitary apoplexy whereby her enlarging pituitary tumour has undergone haemorrhage. This has basically wiped out pituitary function in a moment which explains her cardiovascular collapse and immensely painful headache.

In fact apoplexy can present just like a sub-arachnoid haemorrhage in patients with known adenomas:

Splitting headache - INCREDIBLY BAD

  • Xanthochromia (from extravasation of blood through the diaphrgam sellae into the sub aracahnoid space)
  • Vomiting
  • Loss of consciousness.
109
Q

A woman developed pre-eclampsia during her first pregnancy, she also required syntocin (oxytocin) and several units of blood during the birth. 4 months after the birth she still had not resumed menstruation and felt generally tired. She also noticed loss of pubic hair.

What is the most likely diagnosis?
A. Adrenal adenoma
B. Addisonian crisis
C. Sheehan's syndrome
D. Tuberculosis
E. Addison's disease
F. Pituitary apoplexy
G. Cushing's disease
H. Nelson's syndrome
I. Conn's Adenoma
J. Ectopic ACTH producing tumour
K. Drug withdrawal
L. Diabetes Insipidus
A

C. Sheehan’s syndrome

Pre-eclampsia (proteinuria + HTN in pregnancy) is a major risk for post partum haemorrhage: in this lady’s case she had to receive blood products and Syntocin (a synthetic oxytocin drug used to combat uterine atony and help stop haemorrhaging).

Blood loss doesn’t normally cause changes in pituitary function however in pregnancy the pituitary is particularly vulnerable.

Hyperplasia and Hypertrophy of the lactotrophe cells in pregnancy ( due to increased LH/FSH) leads to an increase in size of the adenphypophysis, however there is no such increase in the vascular supply to the anterior pituitary. When blood is lost in PPH the ant. pituitary thus infarcts and necroses. This is Sheehan’s syndrome…it can present insidiously or in a major way. It may be picked up if the mother fails to breast feed, but amennorhoea following birth and loss of pubic and axillary hair is also a common presentation. Only occasionally is full blown Sheehan’s encountered where the Mother becomes hypothyroid and can develop Diabetes mellitus due to wildly deranged Ant. Pituitary function.

DX: low oestradiol and pituitary hormone levels.

NB: Diabetes Insipidus is a rare complication since the posterior pituitary has a rich arterial blood supply that is rarely compromised in PPH.

110
Q

An overweight 60 year old lady is seen in clinic complaining of tiredness and general weakness, her eyesight is also suffering, she says she can’t see the room if she looks at a book and it’s getting worse. Physical examination reveals multiple scars on her abdomen, they are noted to be dark, in stark contrast to her purple striae. The notes show the patient was on metyrapone and aminoglutethamide before her surgery.

What is the most likely diagnosis?
A. Adrenal adenoma
B. Addisonian crisis
C. Sheehan's syndrome
D. Tuberculosis
E. Addison's disease
F. Pituitary apoplexy
G. Cushing's disease
H. Nelson's syndrome
I. Conn's Adenoma
J. Ectopic ACTH producing tumour
K. Drug withdrawal
L. Diabetes Insipidus
A

H. Nelson’s syndrome

Worsening loss of peripheral vision infers pituitary adenoma.

Hyperpigmented scars suggsts a state of increased ACTH production however this patient doesn’t seem to be in adrenal failure, she is overweight.

She has had abdominal surgery and was on meyrapone ( a drug which inhibits cortisol production) and Aminoglutethamide (an adrenolytic). Basically this woman is overweight and has striae because she was cushingoid: the cause was adrenal hyperplasia. She was on medication to control this but the decision was made to operate and a bilateral adrenalectomy was performed. The consequences f this is that she has very minimal cortisol production and her pituitary is striving to fuel this by pumping out lots of ACTH (hence the dark scars). The visual field problems are due to an undiagnosed pituitary adenoma that was non functional but since her surgery the pituitary tumour has grown substantially since there is no negative feedback coming from circulating cortisol. This is basically Nelson’s syndrome.

NB: these days bilateral adrenalectomy is very rare and the pituitary is sometimes irradiated to prevent Nelson’s from occurring. This patient needs glucocorticoid replacement and transsphenoidal surgery.

111
Q

An obese lady has a low dose dexamethasone suppression test and is found to have hypercortisolism. High dose dexamethasone fails to suppress her cortisol. There is darkening of her palmar creases and she is found to be hypokalaemic. A side observation is dark velvety patches of axillary skin.

What is the most likely diagnosis?
A. Adrenal adenoma
B. Addisonian crisis
C. Sheehan's syndrome
D. Tuberculosis
E. Addison's disease
F. Pituitary apoplexy
G. Cushing's disease
H. Nelson's syndrome
I. Conn's Adenoma
J. Ectopic ACTH producing tumour
K. Drug withdrawal
L. Diabetes Insipidus
A

A. Adrenal adenoma

This lady is cushingoid: that’s a given, but is the pathology Cushing’s disease or syndrome? The dexamethasone suppression test tells us this.

Administration of low dose dexamethasone (0.5 mg) would suppress cortisol production in a normal person. If suppression was not achieved then hypercortisolism is present.

The high dose dexamethasone test uses 2mg which is capable of producing partial/complete suppression of cortisol if the problem is a pituitary tumour (since the pituitary retains some feedback control) However an ectopic source of ACTH would not be responsive to exogenous feedback and so ins not suppressed.

This lady therefore has Cushing’s Syndrome caused by an ectopic source of ACTH. The darkening of the palmar crease confirms excess ACTH and the hypokalaemia supports this showing there is excessive aldosterone stimulation.

Common causes for ectopic ACTH = Small cell carcinoma of the lung and Carcinoid tumours.

The presence of Acanthosis nigricans (velvet under axilla) might push you towards investigating for carcionoid tumours.

NB: Acanthosis is not only caused by Diabetes and in real life is frequently followed up to screen for Ca. EMQS nigricans + Diabetes pretty much

112
Q

An anxious man walks into clinic he is sweating and looks red, his blood pressure is raised, he faints when he is told the dr would like to obtain a blood sample.

What is the most likely diagnosis?
A. Autonomic neuropathy
B. Prader Willi syndrome
C. Insulinoma
D. Grave's disease
E. Phaechromocytoma
F. Essential Hypertension
G. Inconclusive sample
H. Panic attack
A

H. Panic attack

113
Q

A young girl complains of weight loss and headaches, the dr proceeds to perform an abdominal examination during which she becomes anxious flushed and tachycardic.

What is the most likely diagnosis?
A. Autonomic neuropathy
B. Prader Willi syndrome
C. Insulinoma
D. Grave's disease
E. Phaechromocytoma
F. Essential Hypertension
G. Inconclusive sample
H. Panic attack
A

E. Phaechromocytoma

114
Q

A 40 yo man is in for blood tests he brings in a 24 hr urine sample for VMA which is raised. The man eats ice cream and cream cakes for breakfast lunch and dinner.

What is the most likely diagnosis?
A. Autonomic neuropathy
B. Prader Willi syndrome
C. Insulinoma
D. Grave's disease
E. Phaechromocytoma
F. Essential Hypertension
G. Inconclusive sample
H. Panic attack
A

G. Inconclusive sample

115
Q

A clonidine suppression test is performed on a young man complaining of occasional palpitations and anxiety, his plasma catecholamines are suppressed. However his symptoms persist until he eats some of his jelly babies.

What is the most likely diagnosis?
A. Autonomic neuropathy
B. Prader Willi syndrome
C. Insulinoma
D. Grave's disease
E. Phaechromocytoma
F. Essential Hypertension
G. Inconclusive sample
H. Panic attack
A

C. Insulinoma

116
Q

A young girl is seen in clinic. She appears to be uncomfortable and stares at the doctor. She complains of weight loss and palpitations. She is in shorts though the heating is on in the consultation room.

What is the most likely diagnosis?
A. Autonomic neuropathy
B. Prader Willi syndrome
C. Insulinoma
D. Grave's disease
E. Phaechromocytoma
F. Essential Hypertension
G. Inconclusive sample
H. Panic attack
A

D. Grave’s disease

117
Q

A woman has suffered with hypothyroidism for 20 years, she still complains of feeling cold and a bit sluggish.

Each of these patients has been found to have raised blood prolactin, select the most likely aetiology for each case. 
A. Metoclopramide
B. Acetaminophen
C. Inadequate treatment
D. Macroadenoma
E. Microadenoma
F. Non-epileptic seizure
G. Epileptic seizure
H. Ibuprofen
A

C. Inadequate treatment

In hypothyrodisim if it is primary, ( which the majority of cases are) both the TSH and TRH (thyrotrophin releasing hormone) will be high due to loss of negative feedback. Throxine replacement is the normal treatment, however this patient still has hypothyroid symptoms which would suggest she is not receiving enough T4 to be euthyroid. The consequence of this is that her TSH and TRH are probably not suppressed to normal levels. TRH stimulates prolactin production therefore long term hypothyroid patients can develop hyperprolactinaemia. Remember to treat the cause: if the cause is a prolactinoma, management centers around dopamine agonists (Cabergoline or Bromocriptine) and/or surgery.

118
Q

A 21 yo man presents in A+E with prolactin levels raised 10-fold having been found collapsed on the street, witnesses say he was shaking and rigid.

Each of these patients has been found to have raised blood prolactin, select the most likely aetiology for each case. 
A. Metoclopramide
B. Acetaminophen
C. Inadequate treatment
D. Macroadenoma
E. Microadenoma
F. Non-epileptic seizure
G. Epileptic seizure
H. Ibuprofen
A

F. Non-epileptic seizure

This young man seems to have a history suggestive of an epileptic seizure. However when investigating seizures it is important not to jump straight to epilepsy as there are several metabolic and psychogenic situations that can cause seizures which mimic epileptic fits. Deranged sodium and hypoglycaemia can cause fits as can panic attacks. In most seizures of organic cause the cortisol is usually raised (acute stress response). Prolactin levels however do not change with all seizures, in tonic clonic seizures and temproal lobe epilepsy the post ictal prolactin is usually raised at least 3-fold (it can be as mucha as 25 times baseline value). If somebody has a history suggestive of epilepsy and their prolactin is raised above 3 fold it is almost certain that their seizure was epileptic.

119
Q

A 45 year old lady complains of low libido and some galactorrhoea.

Each of these patients has been found to have raised blood prolactin, select the most likely aetiology for each case. 
A. Metoclopramide
B. Acetaminophen
C. Inadequate treatment
D. Macroadenoma
E. Microadenoma
F. Non-epileptic seizure
G. Epileptic seizure
H. Ibuprofen
A

E. Microadenoma

There are no symptoms to suggest space occupying effects of a macroadenoma so it is most likely this lady has a microadenoma (<1cm). high levels of prolactin cause hypooestrogenism and counteract dopamines effect on arousal thus accounting for the lowered libido.

120
Q

A 45 year old lady complains of headache and double vision.

Each of these patients has been found to have raised blood prolactin, select the most likely aetiology for each case. 
A. Metoclopramide
B. Acetaminophen
C. Inadequate treatment
D. Macroadenoma
E. Microadenoma
F. Non-epileptic seizure
G. Epileptic seizure
H. Ibuprofen
A

D. Macroadenoma

Double vision and headaches suggest that the cause of the high prolactin is large and is compressing structures areound the pituitary notably the optic chiasm. Out of the options it is most likely she has a macroadenoma (>1cm) due to these space-occupying effects.

121
Q

A man has a long history feeling unwell having had several bouts of gastritis recently. He has noticed he has had problems getting it up, he has had a number of medications prescribed for the gastritis.

Each of these patients has been found to have raised blood prolactin, select the most likely aetiology for each case. 
A. Metoclopramide
B. Acetaminophen
C. Inadequate treatment
D. Macroadenoma
E. Microadenoma
F. Non-epileptic seizure
G. Epileptic seizure
H. Ibuprofen
A

A. Metoclopramide

With gastritis nausea and vomiting are prominent symptoms, to manage this this gentleman has been prescribed an anti-emetic: metoclopramide. This works partially by antagonising dopamine in the chemotactic trigger zone. It is this antagonism of Dopamine (which normally inhibits prolactin release) which can lead to hyperprolactinaemia.

122
Q

A man has undergone an anterior resection, the epidural he was supposed to have didn’t site properly so he is on strong opioids post op. He has developed hyponatraemia, he is not oedematous and his urine is dark. The urine osmolality is measured at 600 mOsm.

What is the most likely diagnosis?
A. Diabetes Insipidus
B. Liddle's syndrome
C. Conn's adenoma
D. Addison's disease
E. SIADH
F. Morphine overdose
G. Cushing's disease
H. Ectopic ACTH
A

E. SIADH

Opioids and operative pain are known causes for SIADH. The man is hyponatraemic with concentrated urine. His urine is inappropriately concentrated given his dilutional hyponatraemia. This is caused by excessive production of ADH. Excessive fluid replacement post surgery can exacerbate SIADH. The diagnosis requires absence of hypovolaemia and oedema (and use of diuretics). It also depends on urine osmolality being >500 mOsm and high in sodium. Causes Include Drugs, malignancies (ie.SCC of the lung) CNS disorders including Guillain Barré and head trauma. Chest infections in hospital can lead to SIADH developing. Metabolic diseases can also be responsible.

123
Q

A man has hypertension, his dr starts advising him on low salt diets and prescribes him Spironolactone. The man’s father and grandfather had the same problem.

What is the most likely diagnosis?
A. Diabetes Insipidus
B. Liddle's syndrome
C. Conn's adenoma
D. Addison's disease
E. SIADH
F. Morphine overdose
G. Cushing's disease
H. Ectopic ACTH
A

B. Liddle’s syndrome

This man has Liddle’s syndrome a form of pseudohyperaldosteronism which is caused by overactive eNaC ( epithelial sodium channel) found in the collecting ducts. Liddles is an autosomal dominant condition hence the strong family history. The biochemical picture of hypernatraemia and hypokalaemia mimics that of hyperaldosteronism, normally aldosterone modulates the eNacs activity however in Liddle’s the problem is the channel does not dissociate properly so blockade of aldosterone with spironolactone is ineffective. Direct eNaC antagonists are used: Amiloride or Triamterene in conjunction with a low sodium diet.

124
Q

A young woman has high blood pressure and that hasn’t improved with a low salt diet. She has been complaining of headaches and tingling in her fingers. Her hand twitches when the dr takes her blood pressure again. Dr Trouser takes blood samples and sees the woman has
Na+ = 150 mmol/l K+ = 3.0 mmol/l

What is the most likely diagnosis?
A. Diabetes Insipidus
B. Liddle's syndrome
C. Conn's adenoma
D. Addison's disease
E. SIADH
F. Morphine overdose
G. Cushing's disease
H. Ectopic ACTH
A

C. Conn’s adenoma

This lady has true primary hyperaldosteronism caused by a conns adenoma. Hyperaldosteronism should always be considered in the young hypertensive. Hypokalaemia and hypernatraemia are caused by the aldosterone excess. Headaches are due to the hypertension and hypokalaemia there are also a number of musculoskeletal symptoms that patients complain of.

The tingling can be peri-oral and the twitching was alluding to trousseau’s sign of hypocalcaemia. Trousseau’s sign : ALWAYS an EMQ in the exam, basically in hypocalcaemia there is carpo-pedal spasm when blood pressure cuff is inflated to above systolic pressure. Chvostek’s sign : tapping the zygoma produces facial spasm. the hypocalcaemia is caused because hyperaldosteronism leads to metabolic alkalosis and the raised blood pH makes calcium less available to tissues thus causing a functional hypocalcaemia. Liddles is not an option because there is nil family history of note. Mx = spironolactone and surgery.

125
Q

A tanned looking man comes into clinic complaining of feeling under the weather, his bloodwork shows he is
Na+ = 120 mmol/l K+ = 6.0 mmol/l His urinary sodium is noted to be higher than expected.

What is the most likely diagnosis?
A. Diabetes Insipidus
B. Liddle's syndrome
C. Conn's adenoma
D. Addison's disease
E. SIADH
F. Morphine overdose
G. Cushing's disease
H. Ectopic ACTH
A

D. Addison’s disease

Tanned with hyponatraemia: it must be Addison’s. The hyperkalaemia shows there is mineralocorticoid deficiency..replacement therapy is key as Addisonian crisis may ensue.

126
Q

A man seen in the oncology clinic for radiotherapy has bloodwork showing hyponatraemia. He has been a smoker for 50 years.

What is the most likely diagnosis?
A. Diabetes Insipidus
B. Liddle's syndrome
C. Conn's adenoma
D. Addison's disease
E. SIADH
F. Morphine overdose
G. Cushing's disease
H. Ectopic ACTH
A

E. SIADH

Smoker with radiotherapy: the hyponatraemia is caused by SIADH, the aetiology is paraneoplastic syndrome. He has a bronchial or small cell carcinoma which is producing excess ADH.